Andy Test

Pataasin ang iyong marka sa homework at exams ngayon gamit ang Quizwiz!

10. A 45yo man has a hx of gastric outlet obstruction secondary to chronic ulcer disease and is referred to you for tx. He is 5'9 and wts 135 lbs. He has lost 25% of his body mass over the last 3 months and needs nutritional repletion. What are his calculated body weight and caloric needs?

a. 160lbs/1545 Kcal

6. A 46yo man is brain dead following a MVC. His family agrees to organ donation and he undergoes multiple organ retrieval. His BP was 110/60 mmHg on dopamine at the time of retrieval. The organ were approx flushed in situ with preservation and removed in an expeditious manner. What is the longest optimum time the liver can be kept on ice without significant ischemic injury?

a. 18 hours

10. A 50yo man has hepatitis C-induced cirrhosis. His total bilirubin is 3.2 mg/dL, his INR is 1.8, and his Cr is 1.4mg/dL. Which of the following pts should have higher place on waiting list compared to this pt?

a. A 44yo woman with acute liver failure secondary to Wilson's disease

6. A 42 year-old woman is seen in clinic because of a basal cell carcinoma located o the bridge of her nose. Moh's therapy is recommended. The following statement best describes Moh's therapy for skin cancer:

a. A specialized surgical technique used to excise skin cancers in areas where there is a need to preserve normal tissue.

6. A 62-year-old-man is found to have early Stage I esophageal adenocarcinoma. His past medical history is significant for severe bilateral pulmonary emphysematous disease and is s/p bilateral thoracotomy for lung reduction therapy. His is on chronic home oxygen. The best treatment plan for this patient is:

a. A transhiatal esophagectomy with gastric pull-up

9. A 43yo woman is seen in clinic b/c of chronic pigmentation and mild pain in her medial left calf. She has a prior hx of deep vein thrombophlebitis at age 20 following an appendectomy. There is mild swelling and brown discoloration of the left pretibial area. The most likely sequelae of this problem, if left untreated, is:

a. Ankle ulceration

14. A 24yo woman underwent a double lung transplant for cystic fibrosis one year ago. She has been maintained on an immunosuppressive regimen of tacrolimus, mycophenolate mofetil, and prednisone. Recently she has been experiencing a cough and low-grade fever. A CXR reveals bilateral infiltrates, and further workup and bx shows obliterative bronchiolitis. The most likely dx is:

a. Chronic rejection

6. A 58yo man is seen in clinic b/c of frequent UTIs in the last year. He occasionally passes gas from his urethra. He is otherwise well. A barium enema study done 5yrs age showed sigmoid diverticulosis. What is the next best step in the diagnosis for this pt?

a. Computed Tomography

1. A 20 year old man is brought to the emergency room after attempting suicide by ingesting a bottle of drain cleaner. The patient is delirious and tachypneic and tachycardic, with hypoxia and fever. The initial priority for this patient is:

a. Endotracheal intubation

2. An 83yo man is seen in clinic to discuss further cancer management. He has a hx of rectal carcinoma, tx w/ neoadjuvant chemo/ZRT and surgery 3yrs ago. He has been feeling well and is very active, but there has been a recent increase in his CEA to 21. The initial w/u for recurrent/met carcinoma and 4 hepatic lesions were found. On imaging, MRI and PET scan , these lesions are consistent w/ hepatic colorectal mets and no other lesions are found. What is the next step in this pt's management?

a. Evaluation for surgical resection

8. A 46yo man was admitted to the surgical ICU 3w ago w/ severe pancreatitis. He required intensive IVF resuscitation and was intubated upon admission. He remains on parenteral pain meds and insulin. He is afebrile. Vital signs are BP 130/80, P100, R18. He is making g urine. He was transfused yesterday w/ two units of PRBC b/c of a severe anemia. He remains NPO and cannot tolerate anything by mouth. Attempts at passing a feeding tube beyond the pylorus have failed, and he is receiving TPN. His abd is distended and tender in the epigastrium. CT scan yesterday showed large pancreatic phlegmon w/ assoc narrowing of the duodenum and 3 small acute fluid collections. Most recent blood work shows: glucose 140, amylase 200, hct 29. What is the best indication for surgical intervention in this pt?

a. Gastric outlet obstruction

6. A 65yo man has been dx w/ pancreatic cancer. He is physically fit to undergo a major pancreatic resection. A CTA shows 2.5cm tumor in uncinate process that is not impinging on any major vascular structures, and the tumor is deemed to be resectable. A pancreaticoduodectomy is planned. In addition to the gastroduodenal artery, which of the following arteries must be ligated divided to complete the procedure?

a. Inferior anterior pancreaticoduodenal

10. A 38yo woman is undergoing pre-operative counseling prior to undergoing Roux-en-Y gastric bypass surgery for morbid obesity. She has a history of type 2 DM and sleep apnea. Her BMI is 46 kg/M2 . Without proper supplementation, what is the most likely nutritional deficiency to occur following her Roux-en Y gastric bypass surgery?

a. Iron

10. A 60yo woman w/ chronic hepC is brought to an acute care clinic by her family b/c of increasing confusion. PE identifies jaundice, spider angiomata, and splenomegaly (VASE). Neurologic exam shows the pt to be lethargic; asterixis is present. Which of the following pharmacologic agents is most appropriate for tx of this condition?

a. Lactulose

8. A 63yo woman with repeated episodes of cholecystitis is being considered for cholecystectomy. Past surgical hx includes an open appendectomy 40yrs ago and a hysterectomy 15 yrs ago. When counseling a patient about the risks and benefits between an open and laparoscopic cholecystectomy, the patient should know that:

a. Laparoscopic cholecystectomy can be safely performed in patients who have already had open abdominal surgery

3. A 56yo man w/ hx of HepC infxn is seen in clinic to discuss tx for a 7cm mass seen on u/s of the liver. An MRI of the liver, performed w/ gadolinium, shows a mass in segment 3 w/ arterial enhancement and ring-type enhancement on delayed images. The liver is not cirrhotic in appearance, and a recent bx of the nontumor portion of the liver showed only mild fibrosis. The best tx for this pt is:

a. Liver resection

3. A 65-year-old man is in SICU after undergoing appendectomy and abdominal lavage because of a ruptured appendicitis with diffuse peritonitis. He is noted to have hyperthyroidism. Postoperatively, he develops fever and tachycardia. The diagnosis of thyroid storm in the SICU includes the following features:

a. Nearly nondetectable TSH levels, elevated T3, T4 and glucose levels.

4. A 62yo woman comes to office w/ abd pain and fullness for the past 6 weeks. She was d/c from the hospital 10w ago when she was treated for severe bout of pancreatitis. CT scan now shows a 12 cm cystic structure of the central pancreas, which was not present on prior scans. ERCP shows that the cyst communicates with the main pancreatic duct. What is the next best step in management?

a. Pancreatic cyst gastrostomy

3. A 55yo man presents to your clinic b/c he is worried that he may have an abdominal aortic aneurysm. Which of the following factors is associated with the highest risk for development of an AAA?

a. Personal history of an elective popliteal aneurysm repair

9. A 75 year old man comes into the Emergency room complaining of a fish bone stuck in his throat. With difficulty the bone is removed endoscopically, and the patient is sent home. He returns 2 days later with fever, chills, and chest pain. The most likely cause of the symptom is:

a. Posterior Mediastinum

4. A 32yo woman comes to clinic b/c of mass in her left breast. She is 6mo pregnant and has very large breasts. Her mother had BC at age 62. There is a 3.5cm hard, irregular mass in left breast. A 2.5cm mobile mass is palpable in the left axilla. Both the breast and axillary masses show malignant cells on FNA cytology. Which of these factors is a contraindication to lumpectomy?

a. Pregnancy

3. A 50 year old man is being evaluated because or chronic gastroesophageal reflux symptoms. On endoscopy he is found to have Barrett's esophagus with low grade dysplasia on biopsy. Management of this patient should be to:

a. Repeat endoscopy in 6 months with repeat 4-quadrant biopsy of the Barrett's

2. A 23yo woman is referred by her hematologist for splenectomy for ITP. She has hx of easy bruising and heavy menstrual bleeding. She did not respond well to corticosteroids. A complete w/u including BM aspirate is consistent with ITP. Her plt count is 27K. When is the best time to administer polyvalent pneumococcal vaccine?

a. 2 weeks before splenectomy

3. A 52yo man with CHF secondary to ischemic cardiomyopathy is considering heart transplantation. He is not diabetic, and he has no other chronic medical conditions. If he is compliant with his immunosuppressive therapy, what can you predict the 1yr survival rate will be for his cardiac transplantation?

a. 80%

3. Which one of the following is most appropriate for nonoperative management of splenic trauma?

a. A 4yo girl fell from a swing. Her vital signs are stable. CT scan shows subcapsular splenic hematoma involving 10% of the splenic surface

8. A 62-year-old woman is seen in the office to discuss further management of gastroesophageal reflux that is not well controlled with medications. She wants to undergo surgical relief of her reflux symptoms. Workup confirms reflux disease on pH study with a shortened esophagus on upper GI study. The best surgical approach for treating the patient is:

a. A Collis gastroplasty with a Nissen fundoplication

5. A 53-year-old woman comes to clinic because of dysphagia to liquids and solids for the past three weeks. She has been on proton pump inhibitors for treatment of heartburn. A barium swallow show prominent proximal esophageal dilation with a stricture of the distal esophagus (Birds' beak appearance). Endoscopy shows no distal esophageal lesion. Manometry shows elevated lower esophageal sphincter tone with swallowing. What is the most appropriate surgical treatment for her?

a. A Combined Heller myotomy and Nissen fundoplication**

5. A 68yo woman underwent a subtotal gastric resection for a distal gastric cancer. Her stomach was reconstructed with a gastrojejunostomy. On POD3, large amount of bilious drainage is noted coming from a drain placed in her RUQ (800 mL in the last 24 hrs). The most likely represents which of the following?

a. A biliary leak from the duodenal stump

15. A 8yo boy is scheduled for a kidney transplant from his 18yo brother. Immunologic testing shows a six-antigen match. Which of the following immunosuppresion strategies is most appropriate for this recipient?

a. A combination of agents (i.e. CellCept, tacrolimus, and prednisone)

2. An 80 year old man comes into clinic because of difficulty swallowing solids and frequent regurgitation of undigested food for the past three month. A barium swallow demonstrated a Zenker's diverticulum. The best treatment for this man is:

a. A diverticulectomy with cricopharyngeal and lower esophageal myotomy.

5. An 80 year-old man has a recurrent inguinal hernia. He has an incision close to his groin that he states was from a hernia that was fixed as a young adult. He bring you a copy of the operative report and it is unreadable except for the diagnosis of a Ricter's hernia. What was most likely finding at a time of the first hernia repair?

a. A non-circumferential incarceration of bowel wall

1. A variety of patients were evaluated for surgery. For which one of the following are periprocedural prophylactic antibiotics considered standard care of issue?

a. A patient who requires open repair of an abdominal aortic aneurysm***

8. A 43 yo man sustained a severe head injury when he crashed his motorcycle into an oncoming car. In the ED his vital signs are BP 110/60 mmHg and pulse 60. He is intubated, and has no spont respirations. ICP is 30 mmHg. DTRs are absent. At the scene he had decorticate posturing but in the ED he had a decerebrate response to painful stimuli before being intubated. Pupils are fixed at mid positions. CT scan shows severe diffuse edema. Which of the following clinical criteria best determines the cessation of brain function for organ donation in this pt?

a. Absence of spontaneous respirations

2. A 50 yo woman is at the local restaurant for dinner. Her first plate of the evening is brought out to her. As she sees, smells, and thinks about tasting the food in front of her, acid begins to be secreted in her stomach. This initial phase of secretion is mediated by which of the following compounds?

a. Acetylcholine

10. A 52 yo man presents to your office complaining of constant left leg pain. Which of the following associated sxs is most consistent with PAD?

a. Aching foot pain that is exacerbated with leg elevation (decreased blood supply)

9. A 55 year-old woman is seen in clinic two weeks after completing 5 cycles of chemotherapy for a 6.0-cm cancer of her left breast. The tumor is now 2.0 cm in size. She elects to undergo a lumpectomy. The use of chemotherapy in this setting is described as:

a. Adjuvant.

10. A 45-year-old man with a history of inflammatory bowel disease requiring steroids as been admitted to the ICU following a high-speed motorcycle accident. On his fifth post-operative day following a splenectomy and internal fixation of bilateral femur fractures, he develops hypotension (85/40 mm Hg), which is refractory to 3000 mL intravenous saline bolus and dopamine infusion. He is afebrile. Oxygenation saturation on room air is 96%. Lab studies are notable for Hct of 38%, WBC 8, Na 129 mEq/L and K+ 5.2 mEq/L. Chest computed tomography is normal. The next appropriate therapeutic action would be?

a. Administer hydrocortisone.**

4. A33yo woman is undergoing a laparoscopic splenectomy for ITP. She has not responded to corticosteroids. Just prior to surgery, her plt count is 22K. There is persistent oozing from her trocar sites and w/in her abd as the spleen is being isolated from the surrounding structures. Her vital signs are stable. When is the most appropriate time to transfuse platelets?

a. After the splenic artery has been clamped

1. A 45yo woman comes to clinic b/c of severe pain and swelling behind her right nipple-areolar complex. She relates a hx of previous episode that she treated w/ warm compresses until it resolved w/ a thick, yellow d/c from the nipple. She has no FH of breast cancer. She has a 25yr hx of smoking. PE reveals a painful, distorted, partially retracted nipple w/ a yellow d/c. Definitive therapy requires:

a. Antibiotics followed by excision of the subareolar ducts and fistula tract

4. A 60yo man comes to clinic w/ anorexia and wt loss associated with gnawing epigastric pain when eating for the past 2mo. Upper endoscopy reveals the presence of a 2.5cm Type III gastric ulcer. Rapid urease testing of a gastric tissue bx is negative. Four quadrant bx from the ulcer are negative for malignancy. He undergoes a 6week course of therapy including cessation of ulcerogenic agents. Repeat endoscopy reveals the persistence of the type III ulcer with reduction in size to 2.3 cm Repeat 4-quadrant biopsies are negative for malignancy. Which of the following therapies is most appropriate for this pt?

a. Antrectomy with truncal vagotomy

5. A 21yo man comes to the ED b/c of RLQ pain and nausea of 18hrs duration. He vomited once. He has not eaten in 20 hrs and denies being hungry. On PE, he is exquisitely tender in the RLQ w/ rebound tenderness. His WBC is 14,200; other lab tests are normal. Which of the following is the best management for this pt?

a. Appendectomy

5. A 78yo man is seen int eh office for loss of appetite and abd discomfort for the last 3 days. His temp is 36.8C, HR 95, BP 100/60. His epigastrium mildly tender and he is sent for laboratory testing, after which he goes home. The lab tests reveal: WBC 11.5K, Hgb 16, a new increase in Cr from 0.7 to 1.3, a serum Na of 132, HCO3 of 18, amylase 250. For the pt's follow up the most appropriate plan is to:

a. Ask the office staff to call the patient to the ER

8. A 25yo woman comes to clinic b/c of a 2 week hx of tender lump in her right breast. Her last menstrual period was 3w ago. PE reveals discrete 2.5cm mass int eh UOQ of her right breast. U/s shows smooth, oval 2.5cm hypoechoic mass w/ no internal echoes. The next best step in management is to:

a. Aspirate the lesion

1. A 35yo woman is referred for consideration of cholecystectomy following abdominal ultrasonography that was done to evaluate recurrent UTIs. A recommendation for obs w/o surgery would be made for the following condition:

a. Asymptomatic gallstones (note: gallstone pancreatitis, GB polyp, cholecystitis, choledocholithiasis require surgery)

4. A 60yo man w/ end-stage renal disease has undergone hemodialysis 3x weekly for 10yrs. Over the past 6mo, his serum calcium level has risen from 9.0 mg/dL (normal 8.2-10.2) to 11.2 mg/dL. Adjustment of his dialysis prescription, phosphate binder, and vitamin D dosing does not lower his calcium level. Which one of the following mechanisms most likely accounts for his rising calcium?

a. Autonomous parathyroid gland function

8. A 54yo man was evaluated for bilateral buttock and leg claudication and found to have severe bilateral aorto-iliac occlusive disease. There is no evidence of significant disease distally in his extremities. He is not a diabetic. There is no hx of coronary disease and cardiac stress testing was recently normal. He smokes 1PPD. In addition to smoking cessation, what is the best management for this pt?

a. Bilateral aortobifemoral bypass

5. A 36-year-old woman is seen in clinic to discuss options to decrease her risk for developing cancer. She has a strong family history of breast cancer on her mother's side. Her paternal grandfather died of colon cancer, and a paternal aunt had thyroid cancer. The patient has tested positive for the BRCA1 gene. Which of the following surgical procedures is the most appropriate to decrease her risk for developing cancer?

a. Bilateral mastectomy

3. A 29yo woman comes to clinic b/c of 5w hx of right breast mass that she noted on self-exam. She has no significant factors for the development of breast cancer. On exam there is an area of increased density versus a mass in the UOQ of the right breast. The rest of the exam is unremarkable. The most appropriate next step would be:

a. Breast ultrasound

1. A 63 year-old man is scheduled to undergo a liver biopsy for multiple liver lesions he developed 5 years after undergoing a wedge resection of a gastrointestinal stromal tumor (GIST) of the stomach. The pathology returns hepatic metastases consistent with a GIST primary. To guide therapy, you should request the pathologist also stain the tumor for:

a. C-kit protein

2. A 64yo man is referred to your clinic with ischemic rest pain of the left leg and a nonhealing ulcer of his right great toe. His femoral pulses are nonpalpable bilaterally. He does not have DM. His renal function is normal. Which of the following is most appropriate way to assess his vascular anatomy prior to revascularization?

a. CT angiogram of the pelvis and lower extremities

1. A 44yo man is being evaluated for PUD that has not healed despite empiric triple drug tx for Helicobacter pylori and proton pump inhibitors. Serum gastrin is elevated and abdominal CT scan discloses a 2cm mass in the pancreatic tail. Prior to distal pancreatectomy, which one of the following blood tests is most appropriate?

a. Calcium

10. A 43yo woman is seen in the office to discuss surgical plans for removing a small intestinal tumor. She has had several bouts of otherwise unexplained abdominal pain and bloating for the past 6mo. There is evidence of small bowel tumor on capsule endoscopy. You are discussing the embryologic origins of intestinal tumors with her. You strongly suspect that her tumor has arisen from cells that are part of the APUD (amine precursor uptake decarboxylase) system. Which of the following lesions is your most likely diagnosis?

a. Carcinoid tumor

8. A 62yo man comes to the office b/c of a mass in his right neck that he noticed 4 weeks ago. He has a hx of HTN and had a coronary artery stent 2 years ago. He doesn't smoke but he has chewed tobacco for 45 yrs. He is afebrile. Vital signs are normal. He takes a EB daily. On PE he has poor dentition w/ severe tooth decay and gingivitis. There is a mildly tender ulcer on the posterior lateral tongue on the left. There is a hard 2cm node high in the anterior cervical triangle near the angle of the jaw. A FNA is most likely to show which of the following?

a. Carcinoma

5. A 24-year-old woman develops acute liver failure secondary to acetaminophen overdose. The most common cause of death in this situation is:

a. Cerebral edema

4. A 54yo man w/ known dx of GS presents to the ER w/ abd pain and fever for the past 8 hours. He is oriented to time, place, and person. There is no jaundice. There is tenderness w/ guarding in the RUQ of the abd. Ultrasonography shows stones in the GB and normal caliber bile ducts. The most likely dx is:

a. Cholecystitis b/c pts with cholangitis are often jaundiced**

3. A 62yo man comes to your clinic b/c his wife noticed that his eyes were yellow. Otherwise he is feeling well, able to work, and has stable weight. His PMHx is notable for HTN and mild obesity. The lab tests reveal an Alk Phos of 412, total bilirubin of 7.2, AST 110, ALT 105. The most likely dx is:

a. Choledocholithiasis

7. A 43yo woman presents to your clinic complaining of RUQ pain following a fatty meal. She is of Native American descent. She is married and has 4 children. There is no hx of any hematological disorders. She does not drink alcohol. U/S demonstrates GS. Which of the following molecules is primary responsible for the formation of her GS:

a. Cholesterol

7. A 68yo woman presents to the general surgery clinic with a hx of epigastric pain and nausea w/o vomiting. The pain is burning, constant, and sometimes worse after meals. She underwent antrectomy w/ truncal vagotomy and Billroth I reconstruction for treatment of a nonhealing pre-pyloric ulcer 6mo ago. She has mild epigastric tenderness otherwise her PE is normal. W/u includes an upper endoscopy, which reveals erythematous, friable stomach mucosa, a small ulcer near the gastroduodenal anastomosis, and a pool of bile in the dependent portion of the stomach. Mucosal bx reveal a corkscrew appearance of submucosal vessels. Rapid urease testing is negative. Which of the following drugs would be most helpful for this pt?

a. Cholestyramine

10. A 42yo woman comes to the office b/c of tremulousness and palpitations for the past 3 weeks. She says she feels nervous all of the time. She has otherwise been healthy and takes no meds. She is afebrile. She has exophthalmos. Her pulse is irregular. Her hair is thin and there is pretibial edema. Her thyroid is diffusely enlarged and firm. TSH level is low. What is the most likely mechanism for her condition?

a. Circulating IgG antibody that stimulates TSH receptors on follicular cells

5. A 58yo man presents to clinic for evaluation for wt loss surgery. His BMI is 36 kg/m2 . On w/u, he is found to have poorly controlled DM and severe sleep apnea. In addition, he has stable angina, a recently discovered colon cancer, and a past history of alcohol abuse with cirrhosis but no portal HTN. Which of his comorbidities would be considered a potential contraindication for surgical intervention to assist w/ weight loss?

a. Colon Cancer

4. A 78yo woman is sent to the ED from a local nursing home b/c of abd pain and distension for the past 12 hours. She has dementia and a hx of chronic constipation. A barium enema done 2yrs ago showed a very redundant colon but no other abnormalities. She is now afebrile and vital signs are stable. Her abdomen is very distended with tenderness diffusely to deep palpation. Rectal exam shows a trace amt of stool that is heme negative. An acute abd series now shows dilated colon loops and no free air. Laboratory studies show a WBC of 13K and urinalysis is unremarkable. What is the most likely dx?

a. Colonic volvulus

2. A 20 year-old-man is brought to the Emergency Department following a blunt assault while intoxicated. He is hemodynamically stable and complains of abdominal pain. Mild epigastric tenderness is noted and direct palpation. Focused Assessment with Sonography in Trauma (FAST) is negative. What is the appropriate next diagnostic test to assess for significant abdominal injury?

a. Computed tomography of the abdomen and pelvis

4. 42yo man referred for enlarging reducible inguinal hernia. Hx of cirrhosis following hepatitis C. You are trying to assess the level of risk associated with his liver disease. Components of the MELD score for this patient include bilirubin, INR and:

a. Creatinine (BIC)

5. A 41yo woman complains of recurrent nausea and bloating that does not seem to be related to eating. She has not traveled and has no pets. Her vital signs are stable and she is afebrile. There is mild tenderness in the RUQ. The rest of PE is normal. A RUQ u/s is done and reveals a 12x9x10cm right liver cystic lesion. The cystic lesion is found to have a thick wall and multiple septations within it. Select the correct combo of the most likely dx and most appropriate next step in management:

a. Cystadenoma, resection

2. A 40-year-old man is undergoing an open inguinal hernia repair. After incising the skin and the external layer of subcutaneous fat, you see Scarpa's fascia. If you were to follow Scarpa's fascia medially to the scrotum, it would turn into which structure?

a. Dartos Fascia

7. A 44yo man is seen in the office b/c of abd pain. He has thalassemia major. Vital signs are normal. His spleen is massively enlarged. In addition to decreasing the risk for splenic rupture, what is the most likely benefit of splenectomy for this pt?

a. Decrease transfusion requirements

7. A 46yo woman underwent p donor renal transplantation 6mo ago. She cannot tolerate steroids and she is currently maintained on an immunosuppressive regiment of cyclosporine and alemtuzumab (Campath-1H). What is the MOA of alemtuzumab?

a. Depletes T and B Lymphocytes i. Binds CD52, protein present on surface of mature LCs, but not on stem cells from which these LCs are derived.

2. A 45yo woman is undergoing an upper endoscopy b/c of severe abd pain that is not relieved by an OTC PPI and antacids. She recently has been having frequent diarrhea. She has otherwise been healthy and there is no FHx of chronic or inheritable disease. Endoscopy shows hypertrophic gastric rugal folds and a large antral ulcer and multiple duodenal ulcers. Serum gastrin level is 1200 pg/mL. Where is the most likely place to find her gastrinoma?

a. Descending duodenum

7. 65yo man is admitted to hospital b/c of severe abd pain. He has a hx of recurrent bouts of pancreatitis related to alcohol abuse and has been admitted to the hospital multiple times for pain control. CT scan of abd reveals atrophic pancreas w/ some calcifications consistent w/ chronic pancreatitis. In addition to chronic pain, he is at risk for developing which of the following complications of chronic pancreatitis?

a. Diabetes

3. A 68yo man is being seen in clinic in follow-up of a nonhealing prepyloric gastric ulcer that on four-quadrant bx has histological evidence of signet ring cells. Work-up for this pt's condition should include which of the following?

a. Diagnostic laparoscopy

9. A 52 year old man is referred to you by his primary care physician for a possible hernia. The patient states that when he strains while doing sit ups on the floor he notices a bulge in the epigastric area. A CT scan ordered by family physician shows the entire transversalis fascia has thinned out but is intact. On examination he does sit up and indeed has a bulge in the epigastrium from the xiphoid to just below the umbilicus between the rectus muscles. Based on your exam and CT scan findings, what is the most likely diagnosis?

a. Diastasis Recti

4. A 20yo man sustained a severe head injury when he fell while rock climbing. He is brain dead and his family has agreed to organ donation. His vital signs are BP 80/40mmHg and pulse 110/min. He is intubated on controlled pressure ventilation. His O2 saturation is 100%. Which of the following drugs is most useful in maintaining his BP until organ donation?

a. Dopamine

4. A 60yo woman is referred to your office for evaluation of exercise-induced leg pain. She predictably develops an aching pain in her right calf after walking 1 block. She denies a history of coronary artery disease or diabetes. She takes medication for dyslipidemia and HTN and smokes 1PPD. On exam, she has a normal right femoral pulse and nonpalpable popliteal and pedal pulses. Which of the following would be an indication to intervene for her symptomatic right leg PAD?

a. Due to her leg pain, she is on medical leave from her job as a letter carrier

5. A 58yo man comes to you s/p stomach resection for perforated ulcer in the past. He thinks part of his stomach was resected and reconnected to his jejunum. He takes warfarin b/c of chronic afib. He is currently having difficulty maintaining his INR w/ warfarin and the dosages have ben consistently increased. You believe that this may be due to his surgery and think that he may not be adequately absorbing his warfarin. You surmise this b/c most oral meds are absorbed in the:

a. Duodenum

9. A 29yo woman presents to the ER w/ abd pain, N/V. She is known to have GSs since her last pregnancy 2 yrs ago. Her laboratory results reveal normal hgb, plts, and WBC. The amylase, alk phos, and bilirubin are all elevated. She remains afebrile. After administering IVFs and meds for pain and nausea, what is the most reasonable next step?

a. ERCP w/ sphincterotomy and gallstone removal if lab values do not improve

13. A 48yo heart transplant recipient maintained on cyclosporine, mycophenolate mofetil, and prednisone is being treated for a fungal infection of his fingers with oral fluconazole (Diflucan). The most likely effect will be:

a. Elevated serum creatinine i. Due to nephrotoxicity

10. A 58-year-old man was seen in clinic because of dysphagia for 3 weeks. Barium swallows shows a distal esophageal stricture. He undergoes upper endoscopy, with biopsy showing adenocarcinoma. The most useful diagnostic procedure to T stage this patients tumor is:

a. Endoscopic ultrasound

1. A 67yo man is in the ED w/ abdominal and back pain. 2 weeks ago, he was dx with a AAA 5.6 cm in diam. The AAA was discovered incidentally when he had a CT scan to confirm the dx of a kidney stone. He has severe COPD. What is the best management for this pt at this time and the rationale?

a. Endovascular repair since it is assoc with improved survival compared to open AAA repair for a ruptured AAA.

5. A 19yo woman is seen in clinic for counseling after recently being diagnosed with ileocolic Crohn's disease. She asks about the possibility of developing fistulous disease in the future, and whether this would require surgical intervention if it occurred. Which of the following fistulous complications of Crohn's disease most commonly requires surgical therapy?

a. Enterovesical

7. A 30-year-old woman comes to clinic because of dysphagia to solids for the past month. A barium swallow shows a mass lesion in the mid-esophagus. Upper endoscopy demonstrates a smooth, submucosal lesion. Endoscopic ultrasound confirms a smooth delineated mass in the muscularis layer. The best approach to relieve this patient from her symptoms of dysphagia is:

a. Enucleation of the lesion with reapproximation of the muscle layer

5. A 52-year-old man is in the surgical ward has a temperature of 102.5 degrees Celsius four days after a laparotomy and lysis of adhesions for intestinal obstruction. Prior to surgery he was tread with an NG tube and IV fluids for 5 days but his obstruction failed to resolve. His prior surgical history include an appendectomy (age 10) and traumatic splenectomy (age 24). He has had an uneventful postoperative course, has regained bowel function, and his only oral medication is for incisional pain. His PE is unremarkable except for a swollen right FA w/ a small amt of purulent drainage noted from an old IV site. What is the next best step in management?

a. Excision of the IV site

5. A 64-year-old man comes to the office for a routine 3-month follow-up after elective open colon resection for a primary adenocarcinoma of the cecum, T2N0M0. He has noted a slight amount of drainage from his incision site for some time, otherwise he has had an unremarkable postoperative course. On physical examination there is a small, ulcerated nodule on his midline incision, which is draining serious fluid. A :

a. Excision of the lesion and tract with pathologic analysis

7. A 54-year-old man is referred for a 1.0-cm skin lesion on his upper back that his wife noticed while he was getting out of the shower. His past medical history is unremarkable. He has no other suspicious lesions of lymphadenopathy. The most appropriate next step in diagnosis is:

a. Excisional biopsy with a 1 to 2-mm margin of normal skin.

1. A 67-year-old man presents to the emergency department complaining of excruciating right groin pain. On examination he found to have an incarcerate inguinal hernia. He undergoes an open hernia repair. This incision is taken down through the skin, Camper and fascia. What is the next layer?

a. External oblique aponeuroses

9. A 28yo woman comes to clinic b/c of breast pain. The pain started about 3mo ago and is cyclical and bilateral. Menarche was at age 11. She is nulliparous. She was recently started on OCPs b/c of heavy menstrual bleeding. Her paternal grandmother died of endometrial cancer, and her mother and maternal grandmother both had BC in their 40s, but are alive and well. She has no other complaints and no medical problems. Her exam does not reveal any suspicious findings. Which of the following elements of her hx places her at greatest risk for BC?

a. Family history of breast cancer

10. A 38yo woman is concerned about her risk for needing cholecystectomy after her 72yo mother underwent cholecystectomy for cholecystitis. She takes meds for DM and HTN. She recently was treated for an E coli UTI. She has never been pregnant. Her biggest RF for the development of GS is:

a. Female gender

4. A 25-year-old man comes to the office because of a bulge in his groin. He is otherwise healthy. On physical examination there is a reducible bulge inferior inguinal ligament medially at the top of the thigh. No bulge can be palpated at the external ring. Which of the following is the most likely diagnosis:

a. Femoral hernia

5. A 32yo woman is seen in clinic in follow-up. 2 weeks ago she was dx with HTN and was started on an angiotensin converting inhibitor. She brings a BP diary and her BP over past 10 days was b/t 150/90 and 180/100. Today her BP is 160/100 and symmetric in all extremities. Her PE is normal except for an upper abdominal bruit. Electrolytes, BUN, and Cr were normal during her last visit. Which of the following is the most likely diagnosis?

a. Fibromuscular dysplasia of a renal artery

10. A 60yo woman is being seen in clinic following excision of ICIS of her left breast. Radiation therapy to the breast has been recommended and she wants to know if there are any possible problems related to radiation therapy. Which of the following is an adverse effect of whole breast radiation?

a. Fibrosis

10. A 67 year-old man presents to clinic with a 3-month history of a left neck mass. He has a history of squamous cell cancer of the left cheek removed 2 years ago. He denies any other symptoms and he has never smoked. His physical examination reveals a 2.0-cm firm mass in the mid portion of his left neck; the rest of his examination is unremarkable. A positron emission tomography-computed tomography (PET/CT) scan shows a 2.0 cm PET-avid lesion in the left neck with no other suspicious lesions. The next most appropriate step in management is:

a. Fine-needle aspiration biopsy of the neck lesion.

6. A 23-year-old woman is brought to the Emergency Department after being rescued from a house fire. She ran into a burning room and reached into flames to rescue her cat. There are some superficial burns with redness and blisters on her face. There are deep partial thickness burns on her legs and arms and full thickness burns to her hands and forearms. Her thighs are red and blanch with compression. She is in severe pain. Which area of burn injury is most likely to be the least painful in this patient?

a. Hands

3. A 24yo woman is in clinic in follow-up b/c of small bowel motility dysfunction. She has read a lot of info on the internet and has a number of questions regarding the normal physiology of the small intestine. As part of this discussion, you explain the migrating motor complex (MMC). Which of the following best characterizes the MMC?

a. Helps to prevent stasis and bacterial overgrowth

9. A 33yo woman is brought to the ED by her husband b/c of severe diffuse abd pain that began suddenly 2hrs ago. She has otherwise been healthy. Her only med is oral contraceptive. Her initial vital signs were BP 90/60, P110, R20, and her abd was diffusely tender w/ rebound. Her vital signs improved to 110/70 and P90 after receiving rapid infusion of normal saline. A CT scan shows a 2cm solid hypodense lesion on the surface of the left liver w/ evidence of hemorrhage into the abd. Contrast extravasation is thought to be seen. Select the correct combo of the most likely dx and most appropriate next step in management.

a. Hepatic adenoma, arterial embolization

8. A 54-year-old man comes to your office because of abdominal pain and distention. He has a long history of excessive alcohol consumption. On physical examination he has severe scleral icterus. His abdomen is quite protuberant and mildly tender. He has a large umbilical hernia surrounded by engorged veins. How do you best explain these large abdominal wall veins?

a. High Blood pressure portal vein blood is diverted to the lower pressure veins of the abdominal wall via the paraumbilical veins.

8. A 53yo man presents to your office w/ diffuse diarrhea and a fever. Currently undergoing nutritional replacement via tube feedings b/c of gastric outlet obstruction. The possible causes for his D include either infectious cause or complication of tube feeding. Which of the following indicates that this diarrhea may be result of tube feedings?

a. High osmolarity feedings

6. A 60yo woman comes to clinic following partial mastectomy and sentinel LN bx for stage 1 infiltrating ductal carcinoma of right breast. She had menarche at age 14 and menopause at age 53. She has had no previous breast problems and no FH of breast cancer. She has HTN tx w/ a EB. Arimadex therapy is planned. Which of the following is the best way to follow this pt during the next 3 years?

a. History and physical examination every six months, annual mammogram

6. A 70yo man is admitted w/ mental status changes. CT scan of the head and lumbar puncture results are within normal limits. He underwent right upper lobectomy for squamous cell lung cancer one yr ago. His calcium is 13.0 mg/dL and phosphorus is 3.5 mg/dL. His hypercalcemia will e worsened by the administration of which one of the following?

a. Hydrochlorothiazide

8. A 22 year-old man is in the hospital three weeks after and abdominal gunshot wound that penetrated the spleen, stomach, and proximal small bowel. The internal injuries had been repaired primary and the spleen removed. He has been eating a soft food diet. He now has intermittent fevers associated with chills and sweats and uniculated right subhepatic abscess is noted on CT scan. Vital signs are stable. In addition

a. Image-guided percutaneous drainage

4. A 70-year old man presents with an enlarging, nontender mass in his left thigh. The mass has been present for 4 months and he denies any history of trauma. Physical examination reveals an 8.0-cm firm mass in the anterior compartment of the left thigh. An MRI reveals a heterogeneous mass in the quadriceps muscle suspicious for a possible sarcoma. The next most appropriate step in management is:

a. Incisional biopsy.

3. A 34yo woman seen in clinic b/c of episodic painless rectal bleeding w/ bowel movements for several years. She has otherwise been healthy. There is no family history of colon cancer. Examination shows one internal hemorrhoid, which prolapses slightly through the anal orifice, but reduces spontaneously. The best management at this time is:

a. Increase dietary fiber and fluids and reevaluate in 2-3 months

1. A 69-year-old woman with diabetes and peripheral vascular disease presents to the emergency department with sepsis from a necrotizing soft tissue infection. Her blood sugar is 350 mg/dL. Hyperglycemia has been shown to do which of the following?

a. Increase infectious complications

5. A 30yo man underwent a p donor renal transplantation 3mo ago for ESRD secondary to post-streptococcal glomerulonephritis. He has been on CellCept and prednisone for maintenance immunosuppression. A recent episode of rejection was treated with OKT3 and increased dose of prednisone. His azathioprine dose was increased to decrease his steroid dose. His Creatinine has stabilized but he is now leukopenic. What is the most likely cause of his leucopenia?

a. Increased CellCept dose i. CellCept: Mycophenolate mofetil 1. Inhibits inosine monophosphate dehydrogenase (enzyme that controls rate of syn of GMP in de novo p/w of purine synthesis used in prolif of B and T cells) 2. Usually used in 3 compound regiment: calcineurin inhibitor (ciclosporin or tacrolimus) and prednisone)

9. A 70yo man has massive upper GI bleeding while he is in the surgical ICU, where he is being txd for severe pancreatitis w/ necrosis. Arteriography shows bleeding form the splenic artery into the pancreatic phlegmon w/ a fistula to the stomach. Splenic artery embolization is performed. If the spleen completely infarcts, which of the following would you expect to see on the peripheral blood smear?

a. Increased Howell-Jolly bodies

5. A 37yo man comes to the office b/c of palpitations and tremulousness for the past 2mo. His sxs are relieved by eating. He has a hx of PUD and was treated for H pylori 2yrs ago. He has had IBS since he was a teen, manifested by alt constipation and diarrhea. He takes no meds. His PE is normal. Fasting blood sugar is 40 mg/dL and C peptide level elevated. CT showed 1.5cm mass in head of pancreas that enhances brightly on the arteriogram phase. The mass is easily visible on somatostatin scan and there are no other lesions identified. What is the most likely diagnosis?

a. Insulinoma

7. A 43yo woman is seen in clinic 2w following a partial mastectomy and sentinel LN bx for a 1.8cm invasive ductal adenocarcinoma of the left breast. She is complaining of numbness and tingling on the posterior aspect of her upper left arm. She has no muscle w/n in the left upper extremity and has full ROM in her left shoulder. There is no swelling or edema. The surgical incisions are healing well. Injury to which of the following is the most likely explanation of her sxs?

a. Intercostal brachial nerve

7. A 24yo man is in the surgical ICU for tx of injuries sustained in a MCV 2 days ago. At the time of admission he was stable w/ normal vital signs and a GCS of 15. His main compliant was severe RUQ abd pain. A CT scan is obtained and reveals a grade III liver laceration, rib fxs, and right lower lung contusion. No free air or fluid is noted in the abd. Non-op management was elected. Now, the 2nd HD, he has 3 episodes of hematemesis. On exam his abd is b9 w/o any evidence of peritonitis. What is the most appropriate next step in management?

a. Interventional Radiology w/ selective hepatic artery embolization if a bleeding site is identified.

7. A 30-year-old woman is brought to the ER because of partial thickness burns to her chest, arms, and abdomen when a pressure cooker explodes while she is canning tomatoes. Her wounds are bright red, moist, and blanch on direct pressure and involve 22% of her TBSA. She has severe pain. Which one of the following in the best pain management for this patient?

a. Intravenous narcotics initially, with transition to oral narcotics and non-steroidals.

1. A 48yo man is seen in clinic b/c of wt loss and diarrhea. He has a history of alcoholism and chronic pancreatitis. He has been abstinent from alcohol for the past two years. His only medication is pancreatic enzyme replacement. He has a number of questions about his sxs and the absorptive functions of the small intestine. Which of the following best characterizes small intestinal absorptive function?

a. Is closely linked to Na-coupled nutrient absorption

2. A 72-year-old woman has acute respiratory failure as a result of a head injury. Which intervention will help prevent ventilator-associate pneumonia?

a. Keeping the head of the bed elevated

8. A 35yo woman comes to clinic b/c of diarrhea after meals and progressive wt loss for the past 6mo. She has Crohn's disease and has undergone multiple abd operations for Crohn's disease and episodes of adhesive SBO. Six mo ago she underwent a small bowel resection for recurrent Crohn's disease at a site of a previous small bowel anastomosis. Updated endoscopic and contrast evaluations show no evidence of active recurrent Crohn's disease. Which of the following postsurgical scenarios might be most consistent with short bowel syndrome as an etiology for the pt's symptoms?

a. Less than 60cm small intestine preserved

1. A 42yo woman is in clinic to discuss medical management for her obesity. Her BMI is 44 kg/M2 . Sibutramine is one of the FDA-approved anti-obesity drugs. Which of the following combination of mechanism of action (MOA) and side effects (SE) is characteristic of this medication?

a. MOA: Serotonin and norepinephrine uptake inhibitor; SE: HTN

7. A 49yo man has been sent to you by a PCP w/ presumptive dx of short bowel syndrome b/c of decreasing wt and apparent cachexia. He has multiple active sxs and findings. Which of these tx/findings is major cause of cachexia in a pt w/ short bowel syndrome?

a. Malabsorption

6. A thin 18-year-old man is undergoing an appendectomy. An incision made 2-4 cm above the anterior superior iliac spine and runs parallel to the external oblique of the abdomen. The senior resident asks you to name the incision.

a. McBurney

2. An 8yo boy is brought to the ED by his parents b/c of acute abd pain and vomiting. He had been feeling well until the pain developed suddenly several hours before presentation. The parents mention that a CT scan done a year ago following a fall off a bicycle showed malrotation of the intestine. What is the most likely cause of the pt's symptoms?

a. Midgut volvulus

6. A 75yo man presents with jaundice. On imaging a large stone is identified in the neck of the GB where there is surrounding inflammation that involves the adjacent common hepatic duct. The biliary tree is dilated proximal to this area. The common bile duct distal to this is not dilated. Which one of the following is the most likely diagnosis?

a. Mirizzi's syndrome

9. 30yo man with chronic RF secondary to T1 DM is being evaluated for simultaneous kidney/pancreas decreased donor transplant. Which of the following is the best immunologic test to detect Ags from the cellular arm of the immune response in this pt?

a. Mixed lymphocyte culture i. Determines the HLA class II types

1. A 30yo woman with ESRD is scheduled for renal transplantation. The donor is her identical twin sister. Which of the following describes the best immunosuppression regiment for this pt after transplantation?

a. No immunosuppression because of lack of immune reactivity to donor graft isoantigens

6. A 45yo smoker presents to the ER w/ a 4day hx of projectile vomiting. On PE he is HoTive and tachycardic. He has abdominal fullness, decreased skin turgor, and dry mucus membranes. Peristaltic waves visible in his epigastrium. Treatment for this disorder includes placement of a nasogastric tube and fluid resuscitation using:

a. Normal saline

1. You are currently treating an 85kg, 20yo female who is 5'4" tall for perforated duodenal ulcer and sepsis. This pt is classified as which of the following:

a. Obese-high risk of nutritional deficiency

4. A 40 year old man is referred to the office because of a large type II paraoesophageal hernia that was found on a routine screen chest radiograph. He denies any symptoms of chest pain, dyspnea, abdominal pain, or other GI symptoms. He is otherwise healthy and very active. The most appropriate approach at this time is:

a. Observant management

1. A 5yo girl is seen by her pediatrician for purpuric lesions on her arms and legs following an acute viral upper respiratory illness. Her PE shows few petechiae in addition to the purpuric lesions. Rest of exam is normal. Plt count is 44K. BM aspirate shows slight increase in number of megakaryocytes. What is most appropriate next step in management?

a. Observation

2. A 22yo woman comes to the clinic b/c of right breast mass. She is nulliparous. Menarche was a age 13. There is no FH of breast cancer. She is otherwise healthy and takes no meds. There is a 1.5cm firm, nontender, mobile mass palpable in the UOQ of the right breast. Rest of her exam is completely normal. U/S shows an ovoid solid mass measuring 1.5 cm w/ smooth borders. Needle bx shows cells consistent w/ fibroadenoma. What is the next step in management?

a. Observation

1. A 32 yo woman is being evaluated in the ED following a MVC. She was the driver and was wearing a seatbelt. She skidded off the road and hit a tree. She did not lose consciousness. She has mild abd pain but is hemodynamically stable. A contrast-enhanced CT scan of the abd shows a 12cm mass in the right lobe of the liver w/ progressive peripheral-to-central prominent enhancement and a central hypodense region. There is no extravasation of contrast. The most appropriate recommendation for her would be:

a. Observation only

8. A 21yo man is being evaluated in your trauma bay following a MVC 60min ago. He is stable w/ normal vital signs and a GCS of 15. His main compliant is of severe RUQ abd pain. A CT scan is obtained and reveals a grade III liver laceration, rib fractures, and a right lower lung contusion. No free air or fluid is noted in the abdomen. With respect to his liver injury, what is the most appropriate management?

a. Observation w/ close monitoring

10. A 18yo man is admitted to SICU following surgery for injuries sustained in a MVC. He underwent splenectomy for Grade 5 splenic rupture and open reduction and internal fixation of compound right tibia and fibula fractures. He has multiple fractured ribs on the left and a pulmonary contusion on the left. He is intubated. Vital signs are stable. What is the optimal timing for administering polyvalent pneumococcal vaccine for this pt?

a. On the day of d/c from the hospital

1. A 72yo woman comes to the ED w/ 3 days of abd pain and constipation. Temp is 38.2C, BP 130/80, P 90, R 18. On exam she is moderately tender to palpation in the LLQ w/o rebound tenderness. No mass is appreciated. WBC is 14K. Which of the following is the best choice for managing this pt at this time?

a. Order CT scan of abdomen and pelvis

1. 55yo woman w/ no PMH is brought to the ED by her husband b/c of a 24hr hx of abd pain, N/V. She has hx of HTN, non-insulin dependent DM and PUD and underwent tx for H. pylori 2 yrs ago. She underwent cholecystectomy empirically following a bout of pancreatitis 4 years ago. There were no gallstones. Her meds are glyburide and simvastatin. On exam she is tachycardic and has tenderness in the mid-abd and back. Lab values on admin reveal n WBC and amylase ?250 consistent w/ acute pancreatitis. A CT shows evidence of edematous pancreatitis and pancreas divisum. Which of the following is the most likely cause of her recurrent pancreatitis?

a. Pancreas divisum

10. A 67yo man who has been otherwise healthy is undergoing an operation for bx-proven adenoCa of the pancreas involving the head and uncinate process. A bile duct stent was placed preoperatively b/c severe jaundice and pruritus. Pre-op CT imaging and endoscopic US showed a 2.6cm mass in the head and uncinate process and vascular involvement. During exploration the pancreatic mass is easily palpable and the pancreas is not fixed to the surrounding tissue. A 5mm lesion on surface of right lobe of liver was excised and frozen section confirmed a bile duct hamartoma. There are no other findings. What is the best procedure to do at this time?

a. Pancreaticoduodenectomy (Whipple procedure)

9. A 28yo man presents to the ED w/ hematemesis. He reports a hx of chronic diarrhea and upper abd pain. He is not taking any medications. Endoscopy reveals the presence of both an anterior and posterior duodenal bulb ulcer. A gastrin level is elevated at 1200 pg/mL. Electrolytes are w/in normal limits except for an elevated calcium level of 11.3 mg/dL. Diagnostic imaging modalities are all negative for disease. After confirmatory testing, appropriate management for this patient would include:

a. Parathyroidectomy

6. A 24yo man is seen in clinic following total proctocolectomy and ileoanal pouch anastomosis for familial polyposis. Several of his family members are with him and ask if other sites are prone to malignancy besides the colon in family members affected with the disease. The most common site for an affected relative to develop an extracolonic malignancy is which of the following?

a. Periampullary duodenum

8. A 25yo woman is seen in clinic two weeks after being dx w/ HTN. She was started on hydrochlorothiazide and has been taking it daily. She now complains of diffuse muscle m/n for the past week. Her BP is 120/75 mmHg. Electrolyte results are: Na 142, K 3.2, Cl 98, HCO3 28. The next step in management should be screening for which of the following conditions?

a. Primary hyperaldosteronism

9. A 44yo man is seen in clinic complaining of a tender lump near the anus for one week. Exam shows 3cm fluctuant, erythematous swelling in the right anterior perianal skin, very close to the anal orifice. The best choice for management is to:

a. Proceed to the operating room for exam under anesthesia and abscess drainage

6. A 65yo man with a gangrenous toe undergoes an arteriogram. Multiple stenoses are present throughout the arterial tree. Which factor is associated with the largest decrease in pressure gradient across a stenosis?

a. Radius of lesion

9. A 50yo previously healthy man is found to have a right adrenal mass on CT of the abd performed after a MVC in which he was restrained front seat passenger. He is tender over his left lower costal margin. CT scan is w/in normal limits except for fx of the left 9th and 10th ribs. The adrenal mass is 2.1 cm in greatest dimension, solid, and homogenous. The borders of the mass are well defined and regular. The next step in management for the adrenal mass should be which of the following?

a. Repeat adrenal CT scan

2. An 83yo Asian man w/ a known hx of recurrent common bile duct stones presents w/ fever, jaundice, and RUQ pain. The treatment of this pt includes:

a. Resuscitation, IV Abx, and urgent ERCP to relieve the bile duct obstruction

6. A 26yo woman has a chronic cough and decreased exercise tolerance for the past few months. She has otherwise been healthy and takes no meds. She has two cats. PE is normal. Blood work is normal. A CXR shows bilateral hilar adenopathy. Mediastinoscopy and bx is performed. Pathology shows noncaseating granulomas. What is most likely dx?

a. Sarcoidosis

7. A 48yo man is brought to the ED by his wife with a 48hr hx of abd, pain, obstipation, N/V. Initially he had crampy pain, but for the past 4 hrs it has been steady. He was previously healthy and there is no hx of previous surgery. He does not smoke or drink alcohol. His BMI is 24. Vital signs are: pulse 120b/min; BP 100/70mmHg; R 26/min. His temp is 37 degrees C. His heart is regular with no murmurs and his lungs are clear. His abdomen is distended and diffusely tender to palpation. Bowel sounds are absent. Abdominal films show dilated air-filled loops of small intestine and minimal colonic gas. IVFs are started and a NG tube is inserted with immediate removal of 1200 mL of thick greenish-brown fluid. What is the next best step in management?

a. Schedule an emergent laparotomy

3. A 40yo man is seen in the office in f/u of HTN. He was dx 6mo ago w/ new onset HTN and despite multiple drug therapy, his pressure remains poorly controlled. His PE is normal except for sweaty palms and bilateral fine tremor of his hands. Which one of the following is the most appropriate next test for diagnosis?

a. Serum metanephrines

11. A 48yo man has severe cardiomyopathy associated with ventricular arrhythmias and cardiac failure secondary to cardiac sarcoidosis. He ahs an implantable defibrillator that d/cs several times each day b/c of his arrhythmias, despite his multiple medications. He now has severe pulmonary HTN. His chest radiograph shows a large heart and pulmonary vascular congestion. Which of the following conditions would suggest that he is more suitable for a heart lung transplant rather than an isolate heart transplant?

a. Severe pulmonary hypertension

7. 65yo woman has no flatus or stools for three days. She is awake and alert, afebrile, with normal vital signs. Her abd is firmly distended and tympanitic, but nontender. Plain films show a large air-filled loop of colon interpreted as sigmoid volvulus. Which of the following is the best management at this time?

a. Sigmoidoscopic attempt at reduction

8. A 38yo man presents to the ER complaining of abdominal pain, nausea, non-bilious vomiting, diarrhea, and palpitations. He recently underwent an antrectomy w/ billroth I reconstruction for a nonhealing type I gastric ulcer. He reports that the symptoms began approximately 20mins after drinking his first milkshake since the operation. Which of the following drugs would be most helpful for this pt?

a. Somatostatin

5. A 56yo woman being seen in office in f/u of leg ulcer for past 6mo. Despite abx and optimal local wound care, her ulcer has not healed. Her PMH is significant for RA and HTN. There is no hx of DVT. She takes methotrexate, cefocoxib, hydrochlorothiazide, and lisinopril. She is afebrile. Her vital signs are normal. On PE there is an enlarged spleen. There is a large chronic ulcer on her left shin. Pedal pulses are normal. Lab studies show neutropenia. Her Hct and plt count are normal. What is the next best step in management?

a. Splenectomy

5. A 44yo woman comes to clinic b/c of red skin on the right breast for 10 days. Her last menstrual period was 3 weeks ago and her menarche was age 12. Her first child was born at age 25 and she has had no previous breast problems. She has no FH of breast cancer and no hx of any major medical illnesses. On PE, the skin over the RUO breast is red and edematous w/ minimal tenderness. There are no masses, no nipple d/c or axillary lymphadenopathy. Bx of skin shows breast cancer cells. Which of the following is the stage classification for this BC?

a. Stage III

4. A 70yo woman is being transfused following a LAR for rectal cancer earlier today. 30 mins after starting the transfusion, you receive a phone call from the nurse who states that the pt's temp is 39 degrees. The most appropriate action is to:

a. Stop the transfusion and go to the ward to assess the patient

8. 32yo woman presented to ED with incarcerated ventral hernia. Has ESRD and undergoes hemodialysis MWF. Underwent surgery for perforated duodenal ulcer 2ya. PE: tender, nonreducible mass in central portion of epigastric midline incision .Emergency surgery is planned. Which of the following agents is most likely to produce life-threatening hyperkalemia in this pt?

a. Succinylcholine

3. A 75yo man comes to the office b/c of painless jaundice for 6days and a 20pound wt loss over the past two months. He has HTN treated with lisinopril and hydrochlorothiazide. He has never had surgery. He quit smoking 10 yrs ago, but had a 40pack year history prior to that time. He is quite thin and is profoundly jaundiced. There is a palpable globular mass in the RUQ of the abd that is minimally tender to deep palpation. A CT scan shows dilated intra and extra hepatic bile ducts, a distended GB, and a 2cm hypodense mass in the pancreatic head consistent with the dx of pancreatic cancer. The tumor mass does not appear to involve any vascular structures and there is no evidence of metastatic disease. Which of the following is the next best step in management for this patient?

a. Surgical excision

8. A 36yo woman comes to clinic b/c of recurrent painless bleeding and a tissue protrusion from the anus for the past 18mo. On exam, there are moderately large external hemorrhoids. The internal hemorrhoids prolapse and require manual reduction. The best management at this time is:

a. Surgical hemorrhoidectomy

3. A 64-year-old woman is in the emergency room with signs and symptoms of a bowel obstruction. She has no history of prior abdominal operations. The symptoms/signs most consistent with the diagnosis of an obturator hernia would be:

a. Symptoms of intermittent bowel obstruction

2. A 64 year-old woman recently underwent a wide local excision and sentinel lymph node biopsy for a melanoma of her left calf. The final pathology reveals an ulcerated melanoma that is 1.0mm in depth and the sentinel lymph node is tumor free. The patient's tumor, node, metastasis (TNM) staging is:

a. T1bN0M0

12. A 34yo man is found to have elevated fasting blood sugar and an abnormal glucose tolerance test. He recently underwent deceased donor renal transplant for post-streptococcal glomerulonephritis. His maintenance immunosuppressive regiment is tacrolimus, mycophenolate mofetil, and prednisone. The most likely cause of his glucose intolerance is:

a. Tacrolimus

2. A 50yo man develops HTN, tremors, and n LFTs while being treated for an episode of acute rejection four months after transplantations of a deceased donor kidney for ESRD as a consequence of Type 1 DM. His maintenance immunosuppression regiment has included tacrolimus, mycophenolate mofetil, and prednisone, and the doses of these have been increased. He has also been treated with OKT3. Which of the following is most likely associated with the onset of HTN, tremors, and nLFTsin this pt?

a. Tacrolimus i. Also called FK506 1. Activation of T cell receptor normally increases intracellular Ca, which acts via calmodulin to activate calcineurin. Calcineurin then dephosphorylates the txs factor NF-AT, which moves to nucleus of T cell and increases the activity of genes coding for IL2 and related cytokines. 2. Tacrolimus complexes with FKBP12, and this complex interacts with and inhibits calcineuron, thus inhbiting signal transduction and IL-2 txs. ii. Orthoclone OKT3 (Muromonab-CD3)

10. A 38-year-old woman is being treated in the hospital for burns sustained to her left arm when her sleeve caught on fire when she was burning brush. She has third degree burns of circumferential forearm extending just above the elbow. Which of the following treatments will provide the best functional and cosmetic outcome for this patient?

a. Tangential excision and full-thickness grafting

9. A 25yo man is being evaluated because of maroon stool per rectum. He has otherwise been healthy. Colonoscopy is normal and capsule endoscopy suggests that bleeding is coming from the distal small intestine. The presence of a Meckel's diverticulum as a potential etiology for the bleeding may be able to be identified by which of the following tests?

a. Technetium pertechnetate scan

2. A 65yo man is admitted to the hospital b/c of hematochezia. He had three bowel movements consisting of dark red blood. On admission to the hospital, Hgb is 14.6 g/dL, P 80, BP 142/76, RR 16, and he appears comfortable. Over the next 12 hrs he has no bloody stools; repeat Hgb 12.2 g/dL. He then passes another bloody stool. Vital signs remain stable. The best choice for identification of the bleeding site at the time is:

a. Technetium-labeled RBC scan

9. A 65yo man is scheduled to undergo an 80% subtotal pancreatectomy b/c of a mucinous cystic tumor involving the body of the pancreas. The surgeon explains to him that the body and tail of the pancreas will be removed w/ only the head and uncinate remaining. The pt is concerned about his risk for developing DM after the surgery. Which of the following is most accurate info to share w the pt?

a. The Islets of Langerhans are more abundant in the tail of the pancreas than in the head and uncinate

6. A 35-year-old man is declared brain dead following an isolated gunshot wound. His family agrees to solid organ donation but his in depth medical and social history obtained by the organ donor staff details multiple heterosexual encounters and IV drug use. This drug activity occurred within the previous week. Which of the following best assesses his acceptability for organ donation?

a. The latent HTLV virus status in this patient is documented by enzyme-linked Immunosorbent assay (ELISA) and confirmed by nucleic acid testing (NAT)

6. A 55yo woman w/ known hepC cirrhosis and severe emphysema, requiring continuous O2, is admitted w/ abd pain and distention. W/u reveals ascites and pt is treated empirically for spontaneous bacteria peritonitis and her sxs do improve. Incidentally, on imaging, a 3x2x3cm lesion in the liver is found and is consistent with hepatocellular carcinoma. The patient is referred for eval for hepatic transplantation and was found to be not a candidate due to:

a. The severe pulmonary disease

2. A 26yo woman is seen in the clinic b/c of a neck mass. She has otherwise been healthy. There is a 1.8cm nodule in the right lobe of her thyroid. A FNA cytology shows mainly Hurthle cells. What is the next best step in management?

a. Thyroid lobectomy

7. You are evaluating a 75-year-old man in your office with a moderate left-sided inguinal hernia. His past medical history is significant for laparoscopic cholecystectomy, open radical prostatectomy, and thyroidectomy. Which of the following repair is relative contraindication?

a. Total extraperitoneal laparoscopic repair

5. A 22yo woman comes to the office b/c of a large neck mass. She has no other sxs and has otherwise been healthy. She is afebrile. Vital signs are normal. There is a large mass in the right neck that moves with swallowing. U/s shows a 4.8 cm solid mass in the right lobe of her thyroid. FNA cytology shows papillary carcinoma. Serum T3, T4, and TSH levels are normal. What is the next best step in management?

a. Total thyroidectomy

3. A 40yo man is known to have von Willebrand disease. He undergoes an appendectomy and has ongoing bleeding at the site, which persists despite prolonged application of pressure. The best treatment option for this bleeding at this time is:

a. Transfusion of cryoprecipitate i. Contains: FVIII, fibrinogen, vWF, FXIII ii. Factor VIII is bound to vWF while inactive in circ; FVIII degrades rapidly when no bound to vWF; released from vWF by thrombin. iii. vWF binds to collagen, binds to plt gpIb

4. A 72yo man w/ known advanced cirrhosis is brought to the ED by ambulance after vomiting large quantity of blood. The pt has known portal HTN and has undergone endoscopic banding of esophageal varices in the past. He is admitted to the ICU and medical therapy is initiated. Endoscopy is performed and shows bleeding from the gastric varices which are not able to be treated with banding. Which of the following would be the most appropriate next step in the management of this pt?

a. Transjugular intrahepatic portacaval shunt

10. A 28yo man comes to clinic complaining of episodic bloody diarrhea and rectal urgency, which he has had for 3 yrs. He often has painful cramps just before a bowel movement, and he has lost 25 pounds. He easts a regular diet. There is no Hx of travel. He takes no medications. He is afebrile. Vital signs are stable. He is thin. His abd is soft and mildly tender diffusely. Bowel sounds are hyperactive. Rectal exam shows normal anus and DRE shows no masses and a small amt of bloody mucous. Stool for ova and parasites and enteric pathogens are negative. Colonoscopy shows inflamed mucosa w/ ulcerations and mucous from the dentate line to the transverse colon. The right colon appears grossly normal as does the terminal ileum. What is the most likely diagnosis?

a. Ulcerative colitis

7. A 59yo HTNive man is in the ED with acute onset of severe midscapular "tearing" chest pain. Aortic dissection is suspected. An AKG shows ischemic changes and a chest CT confirms a Stanford Type A dissection. What is the best management for this pt?

a. Urgent operative repair

7. A 26yo woman is seen in the office in preparation for surgery for a thyroid cancer. She feels well and has otherwise been healthy. She takes no meds. Her mother had similar cancer rand also had tumors of her pancreas and adrenal gland. Vital signs are normal. There is a 2cm irregular mass in the right lobe of the thyroid. The remainder of her PE is completely normal. Her serum calcitonin level is elevated. Which of the following should be obtained prior to proceeding w/ surgery?

a. Urinary catecholamine levels

8. A 64 year-old man is seen in clinic because of a chest radiograph revealing a new solitary lesion in his left upper lobe. He has a history of melanoma of his left flank resected 5 years ago. He is otherwise healthy and has no other symptoms. He has never smoked. Physical examination reveals a well-healed scar where the flank melanoma was removed and no evidence of local recurrence. He has no new skin lesions or lymphadenopathy. A positron emission tomography-computed tomography (PET/CT) scan shows a 1.0-cm PET-avid lesion in the left upper lobe with no other suspicious lesions. The next most appropriate step in management is:

a. Video-assisted thoracoscopic (VATS) resection of the lesion.

4. A 23yo man is seen in clinic in f/u of known ileocolic crohn's disease. He is concerned with the potential metabolic side effects of his disease. Which one of the following metabolic side effects is he at most risk for developing?

a. Vitamin B12 deficiency

3. A 74 year-old woman is referred with a biopsy-proven melanoma of her right forearm that is 1.4 mm in depth. She has no family history of skin cancer. She has a history of hypertension for which she takes a diuretic. Her physical examination reveals a 2.0-cm healing incision on her right forearm with no other skin lesions of lymphadenopathy. The next most appropriate step in management is:

a. Wide local excision of the forearm scar with sentinel lymph node biopsy.

10. A 76 year old woman is in the hospital recovering ten days following an emergency laparotomy for a perforated diverticulum. She has a history of pernicious anemia and cirrhosis secondary to chronic hepatitis C. On the fifth postoperative day she developed a wound infection and the skin staples were removed and the wound was packed open. The fascia was intact. She is now afebrile with normal vital signs and tolerating a soft diet. Her abdomen is non-tender and the wound is clean with some granulation tissue. Which of the following places her at greatest risk for incisional hernia formation?

a. Wound infection

4. A 52-year-old woman in the surgical care unit following a motor vehicle crash develops acute kidney injury with ruptured spleen and hemorrhagic shock. Which of the following findings is an urgent indication for renal replacement therapy.

a. Hyperkalemia

6. A 2week old boy is brought to the ED b/c of two days of nonbilious vomiting, lethargy, and low urine output. The boy is somnolent. His abdomen is nontender. Ultrasonography shows a hypertrophied pylorus. The most likely metabolic derangement is:

a. Hypochloremic, hypokalemic, metabolic alkalosis

7. A 17yo girl with anorexia nervosa was admitted to the hospital for total parental nutritional support. Two days after the start of her treatment, she experiences proximal muscle w/n and visual defects. This pt most likely has:

a. Hypophosphatemia

10. A 40yo man comes to the ED with a 4day history of N/V. He has had burning abd, epigastric pain for at least 1 week. He takes a EB for HTN. His temp is 37.5 degrees C, BP on admission is 110/90 mmHg, his pulse is 104, and his RR is 14. He weighs 80 kg. Which of the following lab tests would provide the most assistance in the diagnosis and treatment of this patient?

a. Serum electrolytes

2. A 30yo woman underwent total parathyroidectomy for hyperPTH and 2 hyperplastic adenomas are removed. One half of the right inferior gland was implanted in the sternocleidomastoid muscle. On POD#1, she complains of generalized muscle cramps and she is slightly confused. What is the most likely diagnosis?

a. Severe hypocalcemia

2. 62yo woman with morbid obesity, DM, HTN, CKD (GFR 55mL/min) undergoes elective right hip replacement under general anesthesia. Which of the following is the primary goal of postoperative fluid management in this patient?

a. She should be maintained at euvolemia

6. A 67-year-old man has been under your care for chronic leg ulcer. His wound has been progressing nicely, but over the last few weeks you notice a significant thickening and overturning of the wound edge. The appropriate intervention at this point should be to:

a. Biopsy of the wound edges*

8. A 75-year-old man is scheduled to undergo left inguinal hernia repair with local anesthesia, and sedation. He has a history of hypertension treated with hydrochlorothiazide and lisinopril. He weighs 60 kg. There is a reducible left inguinal hernia. What is the safest maximum volume of 1% lidocaine that can be used for local anesthesia in this man?

a. 40 mL???????? (10,20,30,40,50mL choices) i. Rule of safest volume: 1. Xylocaine (lidocaine): 4.5 mg/kg (350mg) (7mg/kg w/ epi—500mg) 2. Bupivacaine 2.5mg/kg (175mg) (3.5 mg/kg w/ epi)—225 mg) ii. 1% = 10mg/mL 1. 2% is 2g/100mL or 2000mg/100mLÆ20mg/mL

5. A 23-year-old woman is brought into the Emergency Department with third-degree burns involving her entire posterior trunk, half her anterior trunk, her circumferential right thigh, and her circumferential left upper extremity. What is the size of her burn as a %TBSA (percent total body surface area)?

a. 45% TBSA

10. You are in Preoperative Eval clinic seeing pts who are scheduled for surgery. Which of the following pts should have preop ECG?

a. 55yo HTNive man being eval for CEA

9. A 42yo woman is being transitions from PN to EN and she is requiring approx 1900 cals/day/ Nurse caring for her wishes to know when to stop the PN. Her PN can be safely stopped when her EN caloric intake reaches what % of her current PN?

a. 66%

9. A 56-year-old man intensive care unit is being treated for 80% TBSA burns involving his trunk, face, and all four extremities. Allograft skin will be used in order to optimize healing and cover his wounds. Which one of the following statements best describes an allograft?

a. A skin graft taken from a human cadaver

5. A 36-year-old woman is intubated for respiratory failure in the intensive care unit for 8 hours. She sustained a liver laceration and multiple pelvic fractures in a high-speed collision. She did not lose consciousness. She was resuscitated with 8 unites of packed red blood cells and 4 liters of lactated Ringer's solution. Her blood pressure is now 110/70 mm Hg and her pulse is 100 beats per minute. Breath sounds are equal bilaterally, and her and is tensely distended. Arterial blood gas analysis reveals a respiratory acidosis, and serum hemoglobin is 13.2 g/dL. What is the most likely diagnosis?

a. Abdominal Compartment Syndrome

3. A 56-year-old man is seen in clinic preoperatively before a right hemicolectomy for a large sessile polyp locate in the cecum. He has hypertension controlled with hydrochlorothiazide, and lisinopril, and type-2 diabetes mellitus controlled with insulin. In order to be effective for this patient, intravenous prophylactic antibiotic therapy should be:

a. Administered one hour prior to operation

11. A 24-year-old man who works as a carpenter is seen in the Emergency Department because of a deep rusty nail puncture wound to the dorsal surface of the foot. He had a tetanus booster shot six years ago. What is the recommended treatment protocol to prevent tetanus in this man?

a. Administration of 0.5 mL absorbed tetanus toxin. **

7. A 54-year-old man comes to the Emergency Department because of diffuse abdominal pain for three days. He has cirrhosis secondary to hepatitis C and a history of GERD. His temperature is 38 degrees Celsius, BP-110/60 mm Hg, P-100 bpm, and R-18/min. There is mild scleral icterus. His chest is clear bilaterally and heart is regular. His abdomen is distended and there is diffuse tenderness. An abdominal CT scan shows diffuse ascites and no free peritoneal air. A gallstone is also noted as well as sigmoid diverticulosis. A peritoneal tap shows 500 leukocytes/mL and gram stain shows G+ cocci. What is the best management for this pt?

a. Admit to the hospital for intravenous antibiotic therapy

6. A 52-year-old woman is assaulted and sustains a brief loss of consciousness according to bystanders. Upon arrival to the Emergency Department, she opens her eyes to painful stimuli only, has decorticate posturing, and moans incomprehensibly. Her Glasgow Coma Scale score is:

a. Answer: 7 i. EVM: 4,5,6

8. A 75yo woman is 3days post right total knee replacement. She develops left leg swelling and a duplex US confirms a left DVT. She is started on low molecular weight heparin. She beings to develop nosebleeds, bleeding from her IV site, and some bleeding from her wound. Which is the best lab test to assess bleeding in this pt?

a. Anti-Xa activity (not PT or PTT b/c she is on LMWH, which binds to Xa; recall-more controlled and less times to take it, etc)

6. A 45-year-old man presents with acute onset of dyspnea and chest pain. Pulmonary embolism is confirmed by CT angiogram. He has a history of heparin-induced thrombocytopenia. The most appropriate initial management includes administration of :

a. Argratroban i. Direct thrombin inhibitor ii. Prophylaxis for DVT in pts with HIT cc

8. A 70 year old woman is brought into the ICU with respiratory failure after undergoing a sigmoid resection perforate diverticulitis. Which ventilator mode would assume the most work of breathing?

a. Assist control ventilation

6. An 82 yo man undergoes screening colonoscopy. Found to have bx proven adenoCa of sigmoid colon. Medical Hx notable for HTN and dilated cardiomyopathy. Which of following findings is most likely to increase his risk of perioperative cardiac complications?

a. Audible S3 gallop.

7. A 55-year-old man is in the hospital 3 days following an open cholecystectomy for acute cholecystitis. He has been ill at home for 2 days before seeking medical attention and was operated on the fay following admission. During induction of anesthesia his systolic pressure fell from his usual 150-160 mm Hg range to 110 mm Hg and remained at the level for the first 30 minutes of the case. During the procedure the GB was torn and bile spilled into the abdomen. Culture of bile recovered E. coli and Klebsiella species. His fascia was closed and skin left open. His ionized Ca during the procedure was 1.00 umol/L. He was continued on ampicillin-sulbactam that was started b/f the procedure. During his last two days he sequestered 4 and 2.5 L of fluid. His systolic BP has been mostly 140-170 mmHg, pulse 80-90s/min. He was started on a E-blocker the day before surgery and this was continued post op. On morning rounds he is noted to be dyspneic w/ O2 sat 98% on 40% facemask, and a RR of 26-30/min. His BP is 180 mmHg systolic. On exam he is sitting upright. His neck veins are distended, crackles are heard throughout both lungs, and his heart has a gallop. His abdomen is mildly distended w/ bowel sounds. His wound is unremarkable. His extremities are cool, his hands and feet are cyanotic. The principle cause of his hemodynamic state is decreased;

a. Cardiac Contractility

1. A 72 year old man in the intensive care units develops the sudden onset of agitation and hypertension (180 mm Hg systolic) 6 hrs after an open abdominal aortic aneurysm repair. He receives a dose of narcotic analgesic and 10 minutes later he has a blood pressure of 90 mm Hg systolic. Pulmonary artery catheter information at this time : Cardiac Index 1.81/min/m2, CVP 28 mm Hg, PAOP 32 mm Hg, systemic vascular resistance 1500 dynes/sec/cm5 . An EKG shows ischemia. He is still intubated and on a ventilator. His hemodynamic state is best characterized as:

a. Cardiogenic Hypoperfusion

1. A 46yo woman underwent total thyroidectomy for multinodular goiter. On the second post-op morning she complains of circumoral tingling and numbness at her fingertips. The most likely additional finding in this pt will be:

a. Carpopedal spasm

1. A 48-year-old man is brought into the Emergency Department after striking his head against the bottom of a swimming pool after diving into shallow water. No loss of consciousness is reported. He has tenderness of the cervical spine, and marked motor weakness of both arms. He is able to move his lower extremities, though he states subjectively that his legs feel weak. Vital signs are normal. What is most likely diagnosis?

a. Central Cord Syndrome

10. A 29-year-old man is in the ER 90 minutes following a fall from a ladder where he sustained fractures of ribs 9 and 10 on the left and a grade splenic laceration documented by CT scan. No other injuries were found. He has received 2 liters of lactates Ringer's solution since arrival. His pulse is not 125 beats per minute and his blood pressure is 95/52 mm Hg. Urinary output over the last hour is 10 mL, and he appears pale and anxious. His physiologic state is best characterized as:

a. Class III Hemorrhagic Shock Class I II III IV Blood loss 15% 15-30% 30-40% >40% Pulse 100 >120 >140 BP N N p p PP N p (due to VC?) p p RR 14-20 20-30 30-40 >35 Urine output >30 cc/h 20-30 cc/h 5-15 cc/h --- Mental status Slightly anxious Mildly anxious Confused Confused/lethargic Fluid replacement Crystalloid Crystalloid Crystalloid, Blood Crystalloid, Blood Initial Fluid therapy (3-for-1 rule) Initial fluid therapy: 1-2L for adult; 22cc/kg for children 3-for-1 rule

8. A 32-year-old man was admitted to the burn unit 12 hours ago with a 45% TBSA third-degree burn. His entire right leg is circumferentially burned. After 12 hours of Consensus formula resuscitation the nurse calls you with a report of no pulse in the right foot. What is the most appropriate next step in management.

a. Confirm the presence of vascular compromise, and then perform lateral and medial escharotomies of the right lower extremity from the hip to the ankle.

2. A 75-year-old man who resides in a nursing home because of progressive dementia and hemiplegia becomes febrile and is found to have a sacral decubitus ulcer. The base of the wound is gray and covered with and exudate. The wound edges are erythematous. What is the most effective next step to reduce the bacterial load in this wound?

a. Debride necrotic tissue from the wound

1. A 37-year-old woman is being treated for a nonhealing elbow wound following excision of a benign nevus. She has been on home TPN for the past 18 months following resection of most of her small bowel due to mesenteric venous thrombosis. Laboratory evaluation reveals zinc deficiency. The most likely mechanism for her no-healing wound is:

a. Decreased fibroblast proliferation, decreased collagen synthesis, and impaired overall wound strength. i. Vitamin K: 2,7,9,10 carboxylation of glutamate; p syn of clotting factors can lead to hematoma formation and altered wound healing. ii. Vitamin A: increases inflam response, increases collagen syn, and increases influx of macs into wound. iii. Mg is required for protein syn. iv. Zinc is cofactor for RNA and DNA polymerase. v. Lack of any one of these vitamins or trace elements will adversely affect wound healing.

7. A 71-year-old patient with a history of alcohol abuse develops the acute onset of confusion and agitation two days after undergoing a femoral-to-popliteal bypass. His heart rate is 120 bpm and blood pressure 150/80mm Hg. His laboratory tests and computed tomography of the head are normal. The most appropriate next step in his management is administration of:

a. Diazepam

8. A 43-year-old woman is in the intensive care unit for hypotension and respiratory failure. On admission yesterday, her blood pressure was 70/40 mm Hg, pulse 130/min, respirations 26/mins, and temperature 40 degrees Celsius. Over the last 24 hours she received 12 liters of isotonic crystalloid infusion, as well as infusions of nonrepinephrine and vasopressin in an effort to maintain her mean arterial pressure >65 mm Hg. Her chest radiograph shows consolidation in the left upper lobe. Sputum sample and four out of four blood cultures at 24 hrs show G+ cocci in chains. She is receiving broad-spec PCN. Her hgb was 13.6 mg/dL on admission has risen to 15.5 mg/dL. Her WBC that was 3600/mm3 on admission has increased to 12000/mm3 . The principle reason for the increase in hgb is plasma volume depletion from:

a. Diffuse interstitial fluid accumulation

8. A 22-year-old man is in the ER 20 minutes after he was stabbed in the left anterior neck, approximately 2 cm below the angle of the mandible. Blood pressure is 88/56 mm Hg, heart rate is 134 beats per minute, and pulsatile bleeding is noted from the neck wound. The patient is intubated and digital pressure is applied to control the hemorrhage. After intubation, breath sounds are equal bilaterally. What is the next appropriate step in management?

a. Emergent operative neck exploration

3. A 78-year-old man was brought to the Emergency Department after he was struck on the head with a baseball bat during a robbery attempt. He has a brief loss of consciousness, but he was awake and alert when he arrived in the Emergency Department. Thirty minutes later, he becomes agitated with a Glasgow Coma Scale score of 12, and is intubated for airway control. CT scan of brain reveals a convex collection of blood just beneath the right parietal bone. What is the most likely dx?

a. Epidural Hematoma i. Note: acute epidural: fully functional/alert during the lucid period ii. If acute subdural, will not be fully alert/functioning (and CT will look different)

3. A 65 year old man is being admitted to the surgical intensive care unit following an elective aortofemoral bypass for bilateral aortoiliac occlusive disease. The intraoperative course was notable for one episode of hypotension (80 mm Hg systolic) when the aortic clamp was released that lasted for about 10 minutes. He has a history of coronary artery disease with stable angina and a negative stress prior to the surgery. When he arrives in the ICU his BP is 130/80 mmHg, P 95/min, and following data measured and calculated from pulmonary artery catheter placed during the surgery: Cardiac index 2.8 l/min/m2 , CVP 16 mmHg, PAOP 18 mm Hg, SVR 950 dynes/sec/cm5 . His hemodynamic state is best designated as:

a. Euvolemic, normal perfusion

2. A 37-year-old man is admitted to the burn unit 6 hours after sustaining 40% TBSA full-and deep partial thickness burns involving his torso, arms, and thighs. His clothes caught on fire when he was starting a charcoal grill with gasoline. He is intubated and receiving intravenous fluid according to the Consensus formula ((4*wt (in kg))*%TBSA). He is receiving sedation and pain medication. His wounds were cleansed and dressed. Which one of the following is the best choice for preventing burn wound sepsis in this patient?

a. Excision and grafting within 5-7 days of the injury

12. A 48-year-old man comes in the Emergency Department because of a painful left index finger for 3 days. He works as a short order cook at a local diner where he works the breakfast shit. He has a history of type-2 diabetes treated with oral medications. He smokes a pack of cigarettes daily. On Exam, there is erythema and swelling involving the soft tissue adjacent to the nail on the ulnar aspect of his left index finger. The swelling and erythema surround the base of the nail but does not extend beyond the DIP joint. It is very tender but there is no increased pain with active flexion or extension of the distal phalanx. There is no tenderness of the pulp of the distal phalanx. What is the best management for this patient?

a. Excision of the overlying nail plate

3. 28yo G1P0 woman who is 20weeks into her pregnancy presents with 2cm hard, nontender mass in upper outer quadrant of her left breast. First noticed mass month before learned she was pregnant. Been enlarging over last two months. Which of following is most appropriate recommendation for her?

a. Excisional bx of mass

7. A 16-year-old boy is brought to the Emergency Department after being shot in an altercation. He is hemodynamically stable, and physical examination reveals a small caliber gunshot wound just to the left of the umbilicus. A second wound is noted in the left lumbar and paraspinal area. His abdomen is diffusely tender, and a small amount of blood is noted on rectal examination. The next step in his management:

a. Exploratory Laparotomy

7. A 2yo (20?) man comes to the ED with a swollen left leg. Last evening he took a 3hr plane ride to return home from a friend's wedding. Duplex US shows a clot in the left femoral vein. Testing confirms that he has activated protein C resistance (APCR). Which of the following best explains these findings?

a. Factor V neutralization is impaired i. Activated protein C resistance (APCR): acquired or inherited (autosomal dominant) ii. Activated PC (with protein S as cofactor) degrades FVa and FVIIIa. APCR is the inability of protein C to cleave FVa and/or FVIIIa—which allows longer duration of thrombin generation and may lead to hypercoag state. iii. Best known and most common hereditary form = Factor V Leiden (FV cannot be inactivated by activated protein C. 1. MC hereditary hypercoag d/o amongst Eurasians 2. FV fxns as cofactor to allow FXa to activate thrombin. 3. Autosomal dominant, incomplete dominance—variations

5. A 70yo woman is scheduled for a hysterectomy. She states is on no prescription medications but does take a number of OTC supplements. Which one of her medications listed below is associated with the potential for increased bleeding?

a. Garlic

7. 52 yo woman with insulin dependent DM scheduled to undergo mastectomy with sentinel LN bx at 7:30AM. Perioperative glucose management of this pt is best achieved with:

a. Half the usual dose of long-acting insulin the morning of surgery

2. A 40yo man is involved in a MVC and sustains a fractured pelvis and ruptured spleen. He has required transfusion of 5 units of packed RBCs. He is at greatest risk for which of the following infections?

a. Hepatitis B

5. A 45 year-old man comes to the Emergency Department with a complaint of upper abdominal and back pain of 12 hours duration. He was admitted to the hospital 3 weeks ago for alcoholic withdrawal. Today he vomited green looking material several times. He denies hematemesis, diarrhea, bloody bowel movements, and liver disease. He is not diabetic. His temp is 38.6 and BP 90/60 mmHg, pulse 120/min, RR 26/min w/ O2 sat 92% of 4L nasal O2. PE shows agitated middle-aged male who is pale w/ cyanotic fingers. He is disoriented to place and date. Chest is clear, neck veins flat, heart is w/o mrg. His abdomen shows epigastric distention and no scars or bulges. He has no bowel sounds. He has percussion tenderness and involuntary guarding in the epigastrium. His hands and feet are cyanotic and cold. Passage of bladder cath returns 100 cc of dark urine. Labortary data are: Name Today 3 weeks ago Hgb 16.7 gm/dL 13.3 gm/dL WBC 16K mm3 Bands 35% Platelets 96K Sodium 132 mEq/L Potassium 3.2 mEq/L Chloride 95 mEq/L TCO2 20mEq/L Glucose 220 mg/dL BUN 45 mg/dL 22 mg/dL Creatinine 1.8 mg/dL 0.8 mg/dL Bilirubin 1.8 mg/dL ALP 130 mEq/L AST 120 IU/L Amylase 1200 U/L pH 7.28 pCO2 28mmHg pO2 66mmHg Arterial lactice acid 6 mEq/L (normal 90%. Now her BP is 95/60 mmHg and pulse is 110/min. Her abd is distended with no bowel sounds. The extremities are warm with brisk capillary refills. Lab studies: hgb - 10.5, WBC 20K, BUN and Cr have n from 30 mg/dl and 1.5mg/dL to 35 and 1.8, respectively. Total Ca is 6.0 mg/dL and ionized Ca is 0.98 umol/L. A pulmonary artery cath is placed and the following data measured or calculated: Cardiac index - 4.8 l/min/m2 , CVP 18 mmHg, pulmonary artery pressure 35/22 mmHg, PAOP 15 mmHg, SVR 600 dynes/sec/cm5 . Her hemodynamic state is best characterized as:

a. Hyperdynamic Perfusion

10. A 65-year-old woman is in the intensive care unit following emergency surgery for perforated diverticulitis. In the Emergency Department her admission blood pressure was 90/60 mm Hg, pulse 120/min, temperature 38.8 oxygen saturation 95% on 4L of nasal oxygen. Her usual blood pressure is 140/70 mm Hg. She received 3 liters of lactated Ringers solution, ciprofloxacin, and metronidazole, and was taken for emergency sigmoid resection, end sigmoid colostomy, and Hartmann's pouch. During the procedure her systolic was rarely above 100 mmHg despite the adminstration of neosynephrine. She received 6 more liters of lactated Ringers as well as 500cc of hetastarch during the 2hr procedure and make 60 cc of urine. Upon arrival in the ICU, a central venous cath is placed and following data retrieved: MAP 60 on neosynephrine, CVP 8 mmHg, CV saturation 55%. ABG: pH 7.30, PaO2 75 on FIO2 60%-PEEP 7.5 ccH2O , pCOw 36 mmHg Arterial lactic acid: 4.2 umol/L Her hemodynamic state is best characterized as:

a. Hypovolemic Hypoperfusion

5. 68yo man seen in clinic with reducible right inguinal hernia that is uncomfortable at times. Hx of HTN and suffered uncomplicated ant wall MI that was txed with lytic therapy. Remain on EB, statin, and antiplt agent. Regarding hernia repair in this pt, what is the most appropriate way to decrease his risk of cardiac complications?

a. If possible, surgery should be postponed for 6 months following infarction.

9. A 73yo woman is in the recovery room following a right hemicolectomy for a cecal adenocarcinoma. She received preoperative subcutaneous heparin for DVT prophylaxis. It is noted that her abdomen has become progressively distended, her HR has risen to 110/min and her BP has dropped from 120/80 to 95/60 mmHg. The attending surgeon decides to take the pt back to the OR and finds 1500mL of blood in the abdomen. The most likely cause of this bleeding is:

a. Inadequate surgical hemostasis during first operation

9. A 56-year-old man develops acute respiratory failure after a bowel resection for ischemic bowel. With an FiO2 of 50% positive and expiratory pressure (PEEP) 5 cm H2-, his PaO2 is 52 torr. Which of the following would be most likely to improve oxygenation?

a. Increase PEEP**

10. A 22yo man crashed his motorcycle into a tree and is brought to the ED. He was unconscious and intubated at the scene. Two large bore IVs were placed and infusions of 2L of normal saline was administered in route to the ED. On arrival to the ED his BP is 80/40 and his pulse is 130/min. He has a flail chest on his right side. A chest tube was placed and yielded 500mL of blood. His abdomen is tender and a focused abdominal sonography for trauma (FAST) exam shows a lot of fluid in his abdomen. His pelvis is unstable, and he has severe deformities of both femurs and a compound fracture of his right leg. Which of the following best describes the optimal use of blood product transfusion for this pt?

a. Initial transfusion of red cells, FFP, and platelets in a 1:1:1 ratio may improve his outcome

3. A 44-year-old woman is brought to the Emergency Department because of burns sustained in a house fire. She fell asleep while smoking, setting in an upholstered chair on fire. She was rescued by a fireman and paramedics. Upon arrival, she is somnolent, has burns to her face with carbonaceous debris in her nose and mouth and respiratory stridor. There are full-thickness circumferential burns on both arms, forearms and her torso and thighs. Her O2 sat is 98% on 2L of nasal Oxygen. What is the next best step in management?

a. Intubate the patient and administer 100% oxygen (ABCs)

2. A 47yo pt underwent major bowel resection approx 7mo ago. The pt has been doing well, overall, until recently when he has been feeling tired. A CBC is done on this pt and reveals a microcytic anemia. Which region of the GI tract was most likely resected?

a. Jejunum (malabs of iron)

10. A 41-year-old woman comes to the office complaining of a large unsightly scars 6 months after she underwent laparoscopic cholecystectomy for biliary colic and cholelithiasis. Her postoperative course was uneventful, she had no pain. On physical examination two of her trocar incision sites are raised, red, and thick and appear to extend beyond the confines of her original incisions. They are nontender and

a. Keloids

9. An 18 year old man sustains a single stab wound to the left parasternal area. Initial blood pressure upon arrival in the ER is 82/46 mm Hg and heart rate is 121 beats per minute. A left sided chest tube is placed, and 100 mL of blood evacuated. Focused Assessment with Sonography in Trauma (FAST) reveals a large amount of pericardial fluid. During FAST examination, the patients blood pressure on arrival in the ED is 82/46 mmHg and HR is 121 beats/min. A left sided chest tube is placed, and 100 mL of blood evacuated. FAST reveals a large amount of pericardial fluid. During FAST examination, the pt's BP becomes undetectable. What is the next most appropriate step in management? :

a. Left Thoracotomy and cardiac repair

4. A 62yo man is in the ICU following surgery for a shotgun wound to the abd 2 weeks ago. He remains intubated and is being txd w/ TPN. He has developed an n resp quotient and you believe this is result of excessive caloric intake. In order to decide which component of the nutritional formula may have the greatest impact on caloric intake, you need to be able to evaluate the relative impact of each caloric source on total caloric intake. Which of the following sequences represents the nutritional value in kilocals from lowest to highest?

a. Lipids, proteins, carbs

8. A 34yo man is admitted from the ED for surgery for a perforated appendicitis. He has diabetes that he controls w/ oral medications. During the preop workup he is noted to have a severely elevated serum glucose and his serum sodium level is 124 mEq/L. The hyponatremia should not be treated initially b/c with hyperglycemia:

a. Measured serum sodium concentration decreases as a result of dilution in response to the osmolar gradient created by the increase in serum glucose

4. A 38yo woman recovering from a laparoscopic cholecystectomy returns to the office 5days after surgery complaining of RUQ pain. She is mildly jaundiced. Subsequent workup suggests a bile leak, and a percutaneous drain is placed after ERCP is unsuccessful. Over the next 24 hours, almost 600 mL of bile-colored fluid is collected from the drain. The electrolyte composition of the fluid in the drain is closest to:

a. Na+ 130 mEq/L; Cl- 100 mEq/L; K 4.0 mEq/L, Bicarbonate 25 mEq/L

9. A 64-year-old man is scheduled for an elective sigmoid colon resection for biopsy-proven adenocarcinoma. His medical history includes a vessel coronary artery bypass graft and insulin-dependent diabetes mellitus. What is the most appropriate antibiotic management for this patient?

a. Should be administered one hour prior to the incision

4. A 46-year-old man was involved in a workplace accident in which his left calf was crushed in a mechanical printing press. The extrication was prolonged and he was brought into the Emergency Department several hours after the initial injury. Upon arrival, he is normotensive with slight tachycardia. His calf is tensely swollen, and he complains of numbness of the toes. Dorsalis pedis and posterior tibial pulses are present. Plain radiographs reveal no evidence of fracture or dislocation. IVF is started. What is the next appropriate step in his management?

a. Operative Fasciotomy

2. A 48-year-old retired army veteran of the Iraq War has profuse diarrhea 10 days after a sigmoid colon resection and reanastomosis for perforate diverticulitis. He is on a regular diet and is afebrile and has mild diffuse abdominal tenderness but no indication of peritonitis. His stool specimen is bloody and C. Difficile toxin is detected. Initial management should be:

a. Oral Vancomycin Therapy (Vanc-can drink IV, but I think they use metronidazole first b/c vanc selects for resistance)

3. A 33yo man is being treated w/ a 35%TBSA burn over the course of the last 3 weeks. He has now completed all of his skin grafting and no longer appears to have any evidence of sepsis or a need for further direct therapy for his burns. At this point you are interested in identifying whether he's beginning to lay down structural proteins and has turned the tide from catabolism to anabolism. Which of the following lab measurements most likely reflects the return to normal protein anabolism?

a. Pre-albumin

4. A 37-year-old man comes to clinic two weeks following an elective inguinal hernia repair. He has minimal pain controlled with ibuprofen. The incision, which was closed with an absorbable suture, appears to be healing normally. There is minimal swelling and no erythema or ecchymosis. A biopsy of the wound at this time would likely reveal:

a. Predominantly collagen with scarce inflammatory cells i. Three phases of wound healing 1. Inflammatory or substrate (1 day-1week (1-2/3d PMNs; 3-7 macs) 2. Proliferative (1week to 1 month) 3. Maturation or remodeling ii. Proliferative and maturation phase relatively constant regardless of wound type. Begin only when wound is covered by epithelium. iii. Substrate phase (lag phase, inflammatory phase or exudative phase) 1. Cells: PMNs, plts, macs a. PMNs predom. 48 hrs (however, if large # bacteria present (neutropenic pt), infexn will occur. Nphil is NOT critical for wound healing but the macrophage is b. Monocytes enter wound after PMNs, reaching max #s approx 24 hrs later. Macs = main cell for debridement (also involved is activation of TMPPs, which are inactive in normal tissue, but TMMP inhibitor activity falls after injury) i. Experimental wounds that are depleted of macs and monocytes show marked inhibition of FB migration, prolif, and loss of collagen production. ii. Macs secrete TGF-E (stimulates prolif of FBs), IL1 (regulate rapir of damaged tissue, induces fever, promotes hemostasis, enhances FB prolif, activates T cells) 2. Primary wound healing, substrate phase ~4 days. Wound is edematous and erythematous 3. Secondary or tertiary intention, this phase continues indefinitely until wound surface is closed by ectodermal elements iv. Proliferative phase 1. Char by production of collagen in the wound. Less edematous and inflamed than before, but wound scar may be raised, red and hard. 2. Primary cell: Fibroblast-produces collagen. a. Hydroxylation of proline and lysine (Vitamin C, ferrous ion, D-ketoglutarate) v. Maturation phase (remodeling) 1. Char by maturation of collagen by intermolecular cross-linking 2. Wound scar gradually flattens and becomes less prominent and more pale and supple. 3. Time of great metabolic activity 4. No net collagen gain; it's just remodeled 5. 9-12 months

6. A 55yo male with severe symptomatic anemia after GI bleeding is rapidly transfused with 3 units of PRBCs. He begins to experience severe muscle cramps. The most likely finding on an ECG would be

a. Prolonged QT interval secondary to ST segment elongation i. Due to HoCa—occurs after blood transfusion, as result of citrate binding and dilution. Decreases in ionized Ca during transfusion correlate with speed of transfusion and circulating citrate levels. Seen more commonly during transfusion of plasma and plts, which have high citrate concentrations. Citrate added to banked blood to act as anticoag and preserve life of blood. Usually rapidly metabolized by the liver as it is transfused and presents no problem. However, when blood is transfused faster than metabolism of the excess citrate can occur, HoCa results. Citrate is neg charged ion and Ca is positively charged; thus the two ions are attracted to each other. Therefore, transient HoCa can occur with massive admin of citrated blood (as in e/x transfusions in neonates), as Ca ion combine with citrate and are temporarily removed from the circ. Citrate metabolism is hindered in pts with liver disease, shock, and hypothermia. Small children and osteoporotic adults are also at increased risk for citrate/Ca imbalances b/c the tend to have inadequate stores of bone Ca, therefore less able to compensate for declining ionized Ca levels. When citrate intoxication occurs, it may be manifested as circumoral parathesis, muscle tremors, or tetany. HoCa causes: hyperactive DTRs, Chvostek sign, tetany, Trousseau's sign)

6. A 37yo man is admitted w/ several-week hx of abd pain, dehydration and HoTN. He is found to have perforated gastric ulcer and undergoes a partial gastrectomy. He is begun on TPN on post-op day 2 and over the next 3 days develops anemia and resp distress. The most likely cause of his sxs is:

a. Re-feeding syndrome

2. A 59 year old woman is in the intensive car unit following emergency surgery for perforated diverticulitis. In the Emergency Department her admission blood pressure was 90/60 mm Hg, pulse 120/min, temperature 38.8 degrees Celsius, oxygen saturation 95% on 4L of nasal oxygen. Her usual blood pressure is 140/70 mm Hg. She received three liters of lactated Ringers solution, ciprofloxacin and metronidazole, and was taken for emergency sigmoid resection, end sigmoid colostomy and hartmann's pouch. During the procedure her systolic was rarely above 100 mmHg despite admin of neosynephrine. She received 6 more liters of LRs as well as 500cc of hetastarch during the 2 hr procedure and made 60 cc of urine. Upon arrival in the ICU a Central venous cath is placed and following data retrieved: MAP 60 mmHg on neosynephrine, CVP 8mmHg, CV sat 55%, arterial blood gas: pH 7.3, pO2 75 on FIO2 60% - PEEP 7.5 ccH2O, pCO2 36 mmHg. Arterial lactice acid 4.2 umol/L. The principle value of lactic acid measurement for this pt is as a marker of:

a. Response to circulatory resuscitation

3. A 63yo woman had a celiotomy for a recurrent bowel obstruction. One week post-op she develops a high output enterocutaneous fistula. Subsequent evaluation indicated the fistula involves her distal small bowel. The most appropriate fluid for replacing the loss from fistula is:

a. Ringer's Lactate

4. A 25-year-old man is brought to the ER after being involved in a high-speed motor vehicle accident during which his vehicle burst into flames. He is unresponsive, on a back board, with a cervical collar on. His blood pressure is 85/45 mm/Hg, pulse is 133 beats/minute, and he has an obvious third-degree burn to a majority of his body. What is the correct sequence of treatment for his patient?

a. Secure his airway, obtain two large bore IVs, bolus 2L LR, examine the patient CT scan the head and abdomen, calculate the TBSA, start the condense formula, treat the burns with silvadine.

1. A 42-year-old is admitted to the burn unit 8 hours after sustaining 60% TBSA full-and deep partial thickness burns involving his face, upper extremities including his hands, his torso and left thigh in a house fire. He is intubated and receiving intravenous fluid according to the Concensus formula. Which one of the following best describes the most appropriate rehabilitation for his patient?

a. Should be directed to restoring patients to functional status by preventing scar contractures and immobility

1. 72yo man scheduled to undergo robot-assisted prostatectomy for adenocarcinoma of prostate. PMHx unremarkable and takes no meds. Which one of following is best management to reduce risk of thromboembolic complications?

a. SubCut LMW Heparin

3. A 39-year-old woman presents because of a nodule at an incision site. She underwent a cesarean section approximately 5 months ago. On examination she has healed Pfannenstiel incision with a 1.5 cm firm nodule in the center of the incisions. The nodule is excised microscopically reveals a granuloma with giant cell formation with polarize refractive material. The most likely diagnosis is:

a. Suture granuloma

9. 72yo man in clinic to discuss Coronary artery bypass grafting. Questions about risks assoc with operation. The term "informed consent" refers to:

a. The process wherein the pt and surgeon decide on a plan.

9. A 61-year-old male is in the recovery room following elective colon resection for colon cancer. This surgery lasted two hours and there was minimal blood loss and intraoperative contamination. Cefoxitin was administered 30 minutes prior to incision. What is the most appropriate postoperative antibiotic regimen for this patient?

a. There is no need for antibiotics

10. A 79-year-old woman has been in the ICU on a ventilator for exacerbation of COPD for 6 days. Her past medical history includes an endovascular graft for an abdominal aortic aneurysm. She has a central venous line for fluid and medication administration because of lack of peripheral IV access. Blood cultures obtained in response to a 102.7 degree Celsius temperature are positive for E. Coli. The most likely source for the

a. Urinary Tract

4. A 64-year-old man is on surgical ward recovering from a gastric resection for a large duodenal ulcer complicated by a duodenal stump leak, pneumonia and catheter related sepsis. A nasal swab was positive for MRSA. He was in the intensive care unit for 5 weeks and received a variety of intravenous antibiotics including clindamycin, a third generation cephalosporin, metronidazole, and clindamycin. Neomycin irrigant was used to irrigate his peritoneal cavity at the initial surgery. He is now noted to have a decrease in hearing acuity. The most likely antimicrobial agent for this toxicity is:

a. Vancomycin (recall: the AGs as well-peaks and troughs (kidneys))

9. A 20-year-old male driver is brought to the Emergency Department following a motor vehicle crash. He was the driver and was not wearing a seat belt. He is on a backboard with a hand cervical collar in place. He is receiving nasal oxygen at 4 liter/min. He is speaking and complaining of chest pain and shortness of breath. His blood pressure is 90/60 mm Hg, pulse 130/min. He has ecchymosis and abrasions over his right chest. There are no breath sounds in the right chest. His O2 sat is 85%. He has cool, cyanotic extremities. Removal of his neck collar with in-line traction of his head reveals distended neck veins. The principle cause of his HoTN is decreased:

a. Venous return

4. A 43 year old man was admitted to the intensive care unit 36 hours earlier for a diagnosis of severe alcohol induced pancreatitis. He weight 70 kg on admission. His mean arterial pressure throughout this time has been less that 65 mm Hg, His urine output has been less that 30 ml/hour, and his poor respiratory status has resulted in mechanical ventilation. His fluid balance is fifteen liters positive and his abdomen is noted to be tense with an abd pressure of 30 mmHg. The principle cause of his ongoing HoTN is decreased:

a. Venus Return

7. A 27-year-old man comes to clinic with a recurrent umbilical hernia three months after initial repair. His BMI is 26. His diet is high in dairy, breads, and meats and he rarely eats fruits or vegetables. A complete blood count is normal. Before attempting repair of the recurrent hernia, supplementation of which of the following should be started and maintained after surgery?

a. Vitamin C

Chapter 4: Surgical Bleeding: Bleeding disorders, hypercoag states, and replacement therapy in the surgical patient 1. J.R. is a 24yo male who has been in the Trauma ICU for 42 days due to a closed head injury and multiple fxs sustained in a MVC. The pt has been NPO due to obtunded state and is on IVF (NS and glucose). He has also been on multiple IV abx as therapy for multi-site infections. Surgery is again scheduled to stabilize his pelvic fx. Pre-op lab studies show the PT time and PTT time both to be mildly elevated. These same tests were normal when the pt was admitted to the hospital. The most likely explanation for this change in his lab results is:

a. Vitamin K deficiency (2, 7, 9, 10; due to multiple IV abx destroying colonic bacteria responsible for making VitK)

9. A 28yo man is admitted following a motorcycle crash in which he sustained a fractured left tibia, multiple left rib fractures, and a head injury. One week after the injury he is suspected of having SIADH because of dependent edema and a persistent low urine output in spite of normal vital signs . He has a serum sodium of 130 mEq/L, and his Hb and hct have slowly decreased to 9.2 gm/dl and 28% respectively. The most appropriate treatment of the suspected SIADH at this time should be:

a. Water restriction to less than 1000 mL/day


Kaugnay na mga set ng pag-aaral

Midterm Chapters 1-6 Social Psych

View Set

Europe Thinking Spatially and Data Analysis - Europe - Physical Geography

View Set

Convection in the Atmosphere and Wind

View Set

Community Development and Planning Quiz 2

View Set

Chapter 41 Pathophysiology NCLEX-Style Review Questions

View Set